Download as pdf or txt
Download as pdf or txt
You are on page 1of 77

ACTIVE SITE EDUTECH-9844532971

SrSR||1212THTHCLASS
CLASS|NEET
|NEETPHYSICS
PHYSICS: VOL-1|
: VOL-1|

Sr| 12th class|NEET PHYSICS

CHAPTER
3

CURRENT ELECTRICITY
CONTENTS NEET Syllabus
  Introduction Electric Current, Flow of electric
  Electric Current charges in metallic Conductors,
  Electric Currents in Conductors Drift velocity and mobility and
  Ohm’s law their relation with electric current;
  Drift of Electrons and the Origin Ohm’s Law,Electric resistance,V-I
of Resistivity
characteristics, Electrical energy
  Limitations of Ohm’s Law
and power, electrical resistivity
  Resistivity of various Materials
and conductivity.
  Temperature Dependence of
Resistivity Carbon resistors, colour code for

  Combination of Resistors – Series carbon resistors, Combination of


and Parallel Resistors–Series and Parallel,
  Electrical Energy, Power Temperature Dependence of
  Cells, emf, Internal Resistance Resistivity,Cells, emf, Internal
  Cells in Series and in Parallel Resistance, Cells in Series and in
  Kirchhoff’s Laws Parallel, Kirchhoff’s Laws ,
  Wheatstone Bridge Wheatstone Bridge, Meter Bridge,
  Meter Bridge Potentiometer-principle and
  Potentiometer applications

152 | CURRENT ELECTRTICITY |


ACTIVE SITE EDUTECH-9844532971

Sr | 12TH CLASS |NEET PHYSICS : VOL-1|


Where f is the frequency of revolution in Hz.
INTRODUCTION  is the angular frequency in rad/sec
Strength of Electric Current v is linear velocity of the charge q
The strength of electric current is defined as r is radius of the circular path
the rate of the flow of charge through any cross 4. If in a discharge tube n1 protons are moving
section of a conductor.
The instantaneous current is defined by the from left to right in t seconds and n2 electrons
equation, are moving simultaneously from right to left in t
Q dQ seconds, then the net current in any cross section
I  Lt 
t  0 t dt of the discharge tube is
q (n1  n2 )e
Average current i  I (from left to right)
t t
Ampere : If one coulomb of charge passes
through a cross-section of the conductor per here e is the magnitude of charge of electron
second then the current is one ampere. (or) proton.
JG
1coulom b Current Density ( J ) : Current density at a
1 ampere =
1 sec ond point is defined as a vector having magnitude
current is a scalar quantity. equal to current per unit area.
Applications on electric current JG I dI
J  Lt  nˆ
1. If the current is varying with time t, then the s  0 s ds
charge flowing in a time interval from t1 to t 2 If the normal to the area makes an angle  with
is the direction of the current, then the current
t2
dI
q   Idt density is J  ,
t1 ds cos  dI  Jds cos 
2. If n particles, each having a charge q, pass (or)
through a given cross sectional area in time t, JG JJG JG JJG
dI  J .ds i.e., I   J .ds
nq JG
then average current is i = SI unit of J is Am–2
t
3(i). If a point charge q is revolving in a circle of Dimensional formula of J is [ AL2 ]
radius r with speed v then its time period is
Current is the flux of current density.
Relaxation time (  ) :
It is the time interval between two successive
collisions of electrons with +ve ions in the
metallic lattice. The resistance of a conductor is
given by
T  (2 r / v) 2ml
R=
ne 2 A
where n = number density of electrons
e = magnitude of electron charge
(ii).The average current associated with this
m = mass of electron ;  = relaxation time.
revolving charge is
DRIFT VELOCITY :
q  vq
I   fq  q Drift velocity is the average velocity acquired
T 2 2 r
by free electrons inside a metal by the

| CURRENT ELECTRTICITY | 153


ACTIVE SITE EDUTECH-9844532971

SrSR||1212THTHCLASS
CLASS|NEET
|NEETPHYSICS
PHYSICS: VOL-1|
: VOL-1|
application of an electric field which results in Illustration-3:
current. Consider a wire of length 4m and cross-
sectional area 1 mm2 carrying a current of
J I 2A. If each cubic meter of the material
Drift velocity v d  
contains 10 29 free electrons, find the
ne Ane
where, J = I/A is current density average time taken by an electron to cross
the length of the wire.
n is number of free electrons per unit volume
Solution:
e is charge of electron
The drift velocity(vd) is related to relaxation time i 2
 vd   29 6 19
ms 1
eE nAe 10  10  1.6  10
(  ) is vd   = 12.5 × 10–4 ms–1
m
Note :1 The drift velocity of electrons is of the Average time taken by an electron to cross the
order of 104 ms 1 . length of wire
2. Greater the electric field, greater will be l 4
t 
v d 1.25  104 s = 3.2 × 10 s
4
the drift velocity v d  E
3. The direction of drift velocity for electrons Mobility (  ) : Mobility (  ) of a charge carrier
in a metal is opposite to that of electric field (like electron) is defined as the average drift
JG
applied E velocity resulting from the application of unit
Illustration -1: electric field strength.
In a Hydrogen atom, electron moves in an drift velocity |V |
 ;  d
orbit of radius 5 × 10-11 m with a speed of electric field E
2.2 × 10 m/s. Calculate the equivalent
6
Mobility depends on pressure and temperature.
current.
OHM’S LAW :
v
Solution: Current i  f .e  .e For a given conductor, at a given temperature,
2 r
the strength of electric current through it is
2.2  106 directly proportional to the potential difference
= × 1.6 × 10–19
2  5 1011 applied across at its ends.
= 1.12 × 10–3 amp = 1.12 mA. V
Illustration -2: i.e. I  V  I 
; V = IR
R
The current through a wire depends on Where R is electrical resistance of the
time as i  i0   t , where i0  10 A and conductor
  4 A / s . Find the charge that crossed Note :
through a section of the wire in 10 seconds. Ø Ohm’s law is neither a basic law nor a derivable
Solution: one
dq Ø Ohm’s law is just an empirical relation.
i  i0   t ; but i  Ø Microscopically, Ohm’s law is expressed as
dt
 dq  (i0   t )dt J  nev d  J   E where  is the
t 10 10 electrical conductivity of the material.
 t2  Ø The conductors which obey Ohm’s law are
q   dq  q  i0t 
t 0  2  0 called Ohmic conductors or ohmic devices.
= (10i0  50 ) = 300 coulomb Ex : all metals

154 | CURRENT ELECTRTICITY |


ACTIVE SITE EDUTECH-9844532971

Sr | 12TH CLASS |NEET PHYSICS : VOL-1|

Ø For Ohmic conductors V – i graph is a straight


line passing through origin (metals).

Non - Ohmic Circuits :


The circuits in which Ohm's law is not obeyed
(A) Slope of the line (B) Here tan 1  tan  2 are called non-ohmic circuits. The V-I graph is
a curve, e.g. torch bulb, electrolyte,
tan   V / I  R ; So R1  R2 i.e T1  T2 semiconductors, thermionic valves etc. as shown
Ø The substances which do not obey Ohm’s law by curves (a), (b), (c).
are called non-ohmic conductors or non-ohmic
devices.
Ex: Thermistor, Electronic Valve, Semi-
conductor devices, gases, crystal rectifier etc., a)

Ø The V – i graph for a non – Ohmic conductor


is non-linear.

b)

c)

Resistance-Definiton :
The resistance of a conductor is defined as the
ratio of the potential difference ‘V’ across the
conductor to the current ‘i’ flowing through the
conductor.
V
Resistance R 
i
S.I unit of resistance is ‘ohm’
Ø The resistance of a conductor depends upon
1) shape (dimensions) 2) nature of material
3) impurities 4) Temperature
Ø The resistance of a conductor increases with
impurities.

| CURRENT ELECTRTICITY | 155


ACTIVE SITE EDUTECH-9844532971

SrSR||1212THTHCLASS
CLASS|NEET
|NEETPHYSICS
PHYSICS: VOL-1|
: VOL-1|

Ø The resistance of a semi conductor decreases conductor such as length, area of the cross
with impurities. section.
3. Resistivity depends on the nature of the material
Factors Effecting the Resistance of A of the conductor, temperature and impurities.
Conductor 4. Resistivity of any alloy is more than resistivity
1. The resistance of a conductor is directly of its constituent elements.
proportional to the length (l) of the conductor i) R alloys  R conductors ii) metals  alloys
(when area of cross section is constant)
Special Cases :
R1 l1
i.e. R l (or) R  l 1. The alternate forms of resistance is
2 2
l2 l 2 d V m
R l R   2 
For small changes in the length,  V m A d A2
R l
Where d is density of material of conductor
2. The resistance of a conductor is inversely V is volume of the conductor
proportional to the area of cross-section (A) m is mass of the conductor
(When length of the conductor is constant) 2. If a conductor is stretched or elongated or
1 1 drawn or twisted, then the volume of the
i.e., R (or) R  2 ; conductor is constant. Hence
A r
l 2
R1  A2   r22  a. R  R l 2
   V
R2  A1   r12 
V 1 1
For small changes in area (or) radius we have b. R  2  R 2  4
A A r
R A 2r
  l2
R A r c. Interms of mass of the wire R
3. As the temperature increases resistance of m
metallic conductors increases and that of m m
and R 
semiconductors decreases. A2 r 4
Conductance: The reciprocal of resistance 3. For small changes in the length or radius during
(R) is called conductance. the stretching
1 R l R A r
conductance, G  . 2 ;  2  4
R R l R A r
The S.I unit of conductance is mho or Siemen 4. In case of a cuboid of dimensions l  b  h
or ohm-1.
Resistivity: As we know, that the resistance
of the conductor is directly proportional to its
length and inversely proportional to its area of
cross section, we can write
l l l
R R Resistance across AB, RAB 
A A bh
where  is specific resistance or resistivity of
b
the material of the conductor. Resistance across CD, RCD 
Note: lh
1. Resistivity is the specific property of a material h
but resistance is the bulk property of a Resistance across EF, REF 
l b
conductor.
2. Resistivity is independent of dimensions of the If l  b  h , then

156 | CURRENT ELECTRTICITY |


ACTIVE SITE EDUTECH-9844532971

Sr | 12TH CLASS |NEET PHYSICS : VOL-1|

1 l
l  h  Rmax  l
2
 
Rmax  ; R   RA
bh
min
l b Rmin h 2
S.I unit : Siemen / m : (Sm–1)
5. If a wire of resistance R is stretched to ‘n’ times For perfect insulators   0
its original length, its resistance becomes n2 R.
6. If a wire of resistance R is stretched until its For perfect conductors,  is infinity.
Temperature dependence of
1
radius becomes th of its original radius then resistance:
n
its resistance becomes n4R. For conductors i.e metals resistance increases
7. When a wire is stretched to increase its length with rise in temperature
by x% (where x is very small) its resistance R t  R o (1  t   t 2 ) for t > 300oC
increases by 2x %.
8. When a wire is stretched to increase its length R t  R o (1  t) for t < 300oC or
by x% (where x is large) its resistance increases Rt  Ro
 0
 x  2 Rot / C

by  2x  100  .
  R0 = resistance of conductor at 0oC
9. When a wire is stretched to reduce its radius Rt = resistance of conductor at toC and
byx% (where x is very small), its resistance
increases by 4x%. ,  = temperature coefficients of resistance
Illustration -4: If R1 and R2 are the resistances at t1oC and
A rectangular block has dimensions 5 cm
× 5 cm × 10 cm. Calculate the resistance R1 1   t1
t2oC respectively then R  1   t
measured between (a) two square ends and 2 2
(b) the opposite rectangular ends. Specific R2  R1
resistance of the material is 3.5 u 105 : m .  
R1t2  R2t1
Solution:
a) Resistance between two square ends The value of  is different at different
A temperatures.
R1 
A 1  dR 
3.5  10 5  10  10 2 At a given temperature     at t0C
R1   1.4  103  Rt  dt 
5  5  10 4

b) Resistance between the opposite rectangular Ø


A
ends R2 
A
3.5  10 5  5  10 2
R2   1.4  104 
5  10  10 4 Graph shows the variation of resistivity with
Conductivity: Conductivity is the measure temperature for conductors, semiconductors and
of the ability of a material to conduct electric for alloys like manganin and constantan.
current through it. It is reciprocal of resistivity.

| CURRENT ELECTRTICITY | 157


ACTIVE SITE EDUTECH-9844532971

SrSR||1212THTHCLASS
CLASS|NEET
|NEETPHYSICS
PHYSICS: VOL-1|
: VOL-1|
Since the resistivity of manganin and constantan
remains constant with respect to change in t  25.510 C ;
temperature, these materials are used for the
bridge wires and resistance coils. t2  t1  25.510 C  t2  0  25.510 C
Ø The resistivity of manganin and constantan is t2  25.510 C
almost independent of temperature.
Ø Two resistors having resistances R1 and Illustration -6:
R2 at 0o C are connected in series. The The resistance of iron wire is 10 and
condition for the effective resistance in
series is same at all temperatures   5  103 C .If a current of 30A is
R1  R2  R1’  R2’ flowing in it at 20C , keeping the
potential difference across its length
R1  R2  R1 (1  1t )  R2 (1   2t ) ; R11   R2 2 constant, if the temperature is increased
Variation of resistance of some materials
to 120C , what is the current flowing
through that wire ?
Solutions:
R120  R20
B
R20 12020
;
R  10
5  103  120
10  100
 R120  15 ; But V = IR
Here V is constant. Hence,
I120 R20 I120 10
 
I 20 R120 ; 30 15 ;  I120  20 A
Illustration -7:
Variation of Resistivity with Resistance of a wire at temperature t0C is
Temperature: R  R0 (1  at  bt 2 )
Ø If 1 is the resistivity of a material at Here, R0 is the temperature at 00C. Find
temperature t1 and 2 is the resistivity of the the temperature coefficient of resistance
at temperature t is
same material at temperature t 2 ,then Solution:
2  1 1   t 2  t1  1 dR 1
 .  [ R0 (a  2bt )]
Illustration -5: R dt R0 (1  at  bt 2 )
The temperature coefficient of resistance
of platinum is at 0° C.  a  2bt 
   2 
3 1
  3.92  10 K  1  at  bt 
Find the temperature at which the increase Illustration -8:
in the resistance of platinum wire is 10% A silver wire has a resistance of 2.1 8 at
of its value at 00C.
27.5C & 2.7 at 1000C. Determine the
110R1 temperature coefficient of resistance of silver.
Solution: R2   1.1R1 ;   3.92  10 3 K 1
100 Solution:
R 2  R1 1.1 R1  R1 Rt = R0 (1 + )
t  
R1  R1  2.1 = R0 (1+ q 27.5) ......(1)
2.7 = R0 (1+ q 100) ......(2)
R1 (1.1  1) 0.1R1 0.1
   Solve equation (1) and (2) 0 &-1
R
1 1R 3.92  10 3

158 | CURRENT ELECTRTICITY |


ACTIVE SITE EDUTECH-9844532971

Sr | 12TH CLASS |NEET PHYSICS : VOL-1|


themometers which can measure low
Illustration -9: temperature of order of 10K and small
V- I graph of a conductor at temperature –3
changes of in the order of 10 K.
T1 and T2 are shown in the figure (T2 – T1) Ø Having -ve  , these are widely used in
is proportional to measuring the rate of energy flow in micro
wave beam.
Ø Thermistor can also be used to serve as
thermostat.
Resistor Colour codes
Colour Number Multiplier Tolerance(%)

Black 0 × 10°
Solution : Brown 1 × 101
Slope of line gives resistance Red 2 × 10 2
So, R1  tan R  R 0 1 BT1
Orange 3 × 103
R 2  tan(90 R)  cot R  R 0 1 BT2
Yellow 4 × 10 4
Green 5 × 105
cot R  tan R  R 0B T2  T1

Blue 6 × 106
cos R sin R Violet 7 × 107
or   R 0B T2  T1

sin R cos R Gray 8 × 108


cos 2R
R 0B T2  T1
 White 9 × 109 –
sin 2R
Gold – ×10–1  5%
2 Silver – ×10–2  10%
or T2  T1 B cot 2R No color –  20%
Illustration -10:
How many number of turns of nichrome
wire of specific resistance 106 m and
diameter 2mm that should be wound on a
cylinder of diameter 5cm to obtain a
resistance of 40 ? Colour bands on a resistor: B.B.ROY
Solution : of Great Britain having Very Good Wife with
If R is the radius of the cylinder Gold and Silver
r is the radius of the wire Ø Resistors in the higher range are made mostly
N is the number of turns from carbon. Carbon resistors are compact,
inexpensive and thus find extensive use in
A S 2 Q R
N
then R ’  = R ’ electronic circuits.
A Qr2 Illustration -11:
Suppose the colours on the resistor as
106 2q2.5q102qN

40  N  800 shown in figure are brown, yellow, green


1q106 and gold as read from left to right. Using
Thermistor: A thermistor is a heat sensitive the table, find the resistance of the
and non-ohmic device. resistor
Ø This is made of semiconductor compounds as Solution:
oxides of Ni, Fe, Co etc.
Ø This will have high +ve (or) -ve temperature
coefficient of resistance.
Ø Thermistor with -ve ‘  ’ are used as
resistance

| CURRENT ELECTRTICITY | 159


ACTIVE SITE EDUTECH-9844532971

SrSR||1212THTHCLASS
CLASS|NEET
|NEETPHYSICS
PHYSICS: VOL-1|
: VOL-1|

Brown Yellow Green Gold 1. If resistors of resistance R1, R2, R 3 ……..are


1 4 ×105  5% connected in parallel, the resultant resistance
5 R is given by
R  14  105 
100
14  105 
1 1 1 1
    ............
 (1.4  0.07)106   (1.4  0.07)M R R1 R 2 R 3
Some times tolerance is missing from the code 2. If resistances R1 and R2 are connected in
and there are only three bands. Then the
R 1R 2
tolerance is 20%.
parallel, the resultant resistance. R  R  R
1 2
Ø Super Conductor : There are certain
metals for which the resistance suddenly falls 3. When resistors are joined in parallel the
to zero below certain temperature called potential difference across each resistor is
critical temperature. same. But the currents are in the ratio i1 : i2 :
Ø Critical temperature depends on the nature of i3:.............
material. The materials in this state are called 1 1 1
=R R : : : ............
super conductors. 1 2 R3
Ø Without any applied emf , steady current can
be maintained in super conductors. 4. When two resistances are parallel then
Ex: Hg below 4.2 K or Pb below 8.2K IR2 IR1
I1  and I2 
Resistances In Series: R1  R2 R1  R2
5. When resistors are joined in parallel, the
effective resistance is less than the least
resistance in the circuit.
Note:
1. If resistors of resistances R 1, R 2, R3, ..... are 1. A wire of resistance ‘R’ is cut into ‘n’ equal
connected in series, the resultant resistance parts and all of them are connected in parallel,
R = R1 + R 2+ R3 + ........ R
2. When resistances are connected in series, same equivalent resistance becomes 2 .
n
current passes through each resistor. But the
2. In ‘n’ wires of equal resistances are given, the
potential differences are in the ratio
number of combinations that can be made to
V1 : V2 : V3 ..... = R1 : R2 : R3 .....
give different resistances is 2n –1 .
3. When resistors are joined in series, the effective
resistance is greater than the greatest resistance 3. If ‘n’ wires of unequal resistances are given,
in the circuit. the number of combinations that can be made
4. When two resistances are connected in series to give different resistances is 2n (If n >2).
then 4. If R s and R p be the resultant resistances of
VR1 VR2 R1 and R2 when connected in series and parallel
V1  and V2  then
R1  R2 R1  R2
Resistances in Parallel R1 
1
2R s  R s2  4R s R p 
 R   4R R 
1
R2 s R s2 s p
2
5. If a uniform wire of resistance R is,
stretched to ‘m’ times its initial length and
bent into a regular polygon of ‘n’ sides then
a) Resistance of the wire after stretching is
R1  m 2 R (' R ’ l 2 )
160 | CURRENT ELECTRTICITY |
ACTIVE SITE EDUTECH-9844532971

Sr | 12TH CLASS |NEET PHYSICS : VOL-1|

m2 R Illustration -12:
b) Resistance of each side R2  For a circuit shown in fig. Find the value
n
c) Resistance across diagonally opposite points of resistance R2 and current I2 flowing
through R2
 n 
 2 R2  m2 R
R0     R0 
 2  4
 
d) Resistance across one side
(n  1) (n  1)m 2 R Solution :
R3  R2  2 If equivalent resistance of parallel
n n
6. 12 wires each of resistance ‘r’ are connected combination of R1 and R2 is R, then
to form a cube. Effective resistance across R
R1R2

10R 2
5r R1  R2 10  R 2
a) Diagonally opposite corners = .
6 V
According to Ohm’s law, R 
3r I
b) face diagonal  .
4 50 10R2
R  5   5  R  10 .
7r 10 10  R2 2

c) two adjacent corners  . The current is equally divided into R1 and R2.
12
7. If two wires of resistivities 1 and  2 , lengths Hence I2 = 5A.
l1 and l2 are connected in series, the equivalent Illustration -13:
Two wires of equal diameters of
1 l1   2 l 2
resistivity   l  l . resistivities 1 and 2 and length x1 and
1 2
x2 respectively are joined in series. Find
  2 the equivalent resistivity of the
If l1  l2 then   1 .
2 combination.
21 2 Solution :
If l1  l 2 then conductivity  =    . A A
1 2 Resistance, R1  1 1 ; R2  2 2
8. If two wires of resistivities  1 and  2, Areas A1 A2
of cross section A1 and A2 are connected in A 1  x1 , A 2  x2
parallel, the equivalent resistivity
As the wires are of equal diameters
1 2  A1  A2 
 = . A1  A2  A .
1 A2   2 A1
x1 x x
21  2 R1  , R2  2 ; R
A A2 A
If A1 = A2 then  =    .
1 2
where x = x1 + x2; R = R1 + R2
1   2
and conductivity   . x 1x1 2 x2
  ; x  1x1  2 x2
2 A A A
9. If ‘n’ wires each of resistance ‘R’ are connected
to form a closed polygon, equivalent resistance (x1  x2 )  1x1  2 x2 [' x  x1  x2 ]
across two adjacent corners is x1 x 2
1x1  2 x2
 n  1   also 1 T1 T2

Reff   R x1  x2
 n  T x1 x 2

| CURRENT ELECTRTICITY | 161


ACTIVE SITE EDUTECH-9844532971

SrSR||1212THTHCLASS
CLASS|NEET
|NEETPHYSICS
PHYSICS: VOL-1|
: VOL-1|
Illustration -14: Illustration -16:
Find the equivalent resistance of the P and Q are two points on a uniform ring of
network in Fig. between points (i) A and resistance R. The equivalent resistance be-
B and (ii) A and C. tween P and Q is

Solution :
Solution: Resistance of section PSQ
i) The 10 and 30 resistors are connected
in parallel between points A and B. The
equivalent resistance between A and B is
10  30
R1  ohm = 7.5
10  30
ii) The resistance R1 is connected in series with
resistor of 7.5 , hence the equivalent
resistance between points A and C is, R RR
R1  . rR  ; Resistance of section PTQ
R2=(R1+7.5) ohm = (7.5+7.5) ohm = 2Q r 2Q
15  . Rr 2Q R

Illustration -15: R2  ; R 2Q  R

2Qr R2 
Find potential difference between points 2Q
As R1 and R2 are in parallel
A and B of the network shown in Fig. and
distribution of given main current R1R2 RR
through different resistors. So, Req   2 2QR

R1 R2 4Q
Illustration -17:
Determine the current drawn from a 12V
supply with internal resistance 0.5 in
the infinite network shown in fig. Each
resistor has 1 resistance.

Solution :
Between points A and B resistors of 4 , 6
and 8 resistances are in series and these are
in parallel to 9 resistor.
Equivalent resistance of series combination is Solution :
R1 = (4 + 6 + 8 ) ohm = 18 First calculate net resistance of d network
If equivalent resistance between A and B is
R = 9 × 18 / (9 + 18) ohm = 6 
Potential difference between A and B is
V = IR = 2.7 × 6V = 16.2V
Current through 9  resistor = 16.2/9=1.8A
Current through 4  ,6  and 8  resistors =
2.7 – 1.8 = 0.9A.

162 | CURRENT ELECTRTICITY |


ACTIVE SITE EDUTECH-9844532971

Sr | 12TH CLASS |NEET PHYSICS : VOL-1|

x Where s  4200J /Kg0C


x=2+ ; 2 ; where J is mechanical equivalent of heat.
x+1 x  2x  2  0
on solving, x  1 3  2.738 Fuse wire: A fuse wire generally prepared
Total resistance = 2.73 + 0.5 = 3.23 from tin - lead alloy (63% tin + 37% lead). It
should have high resistivity, low melting
12
I= =3.73A point.
3.23
Let R be the resistance of fuse wire.
JOULE’S LAW:
SL
According to Joule’s law, the current passing We know that R 
Qr2
through a conductor produces heat. (L and r denote length and radius)
W = vit The heat produced in the fuse wire is
Now, work done, W = (iR) i t
i 2 SL
2 H  i2 R 
W = i2 R t = v t = v i t Qr 2
R
Ø If H0 is heat loss per unit surface area of the
This work is converted into energy in the fuse wire, then heat radiated per second is =
conductor.
 Thermal energy produced, Q = i2 Rt in H 0 2 rL At thermal equilibrium,
joules i 2S L i2 S
 H 0 2 Q rL H
(or) 0 
i 2 Rt Qr 2 2Q2 r3
or Q = cal According to Newton’s law of cooling.
4.2
As H  i 2 , heating effect of current is H 0  K
common to both A.C and D.C. Where  is the increase in temperature of fuse
Joule’s effect is irreversible. wire and K is a constant.
Electrical Energy: i2S
R 2 3
Ø The electric energy consumed in a circuit is 2Q r K
defined as the total workdone in maintaining Here  is independent of length L of the fuse
the current in an electric circuit for a given time.
wire provided i remains constant.
V2t
Electrical Energy = Vit = Pt = i2Rt  For a given material of fuse wire i 2 r 3 .
R
S.I. unit of electric energy is joule Ø If radiation losses are neglected, due to
1 K.W.H. = 36 × 105 J heating effect of current the temperature
Electrical Power: of fuse wire will increase continuously, and
Ø The rate at which work is done in maintaining it melt in time ‘t’ such that
the current in electric circuit. Electrical power
I 2 Rt
W V 2 H  ms  ;  ms(mp   r )
P  Vi  i2R  watt (or) joule / sec J
t R
Ø Heat energy produced due to the electric  2 r 4 s  mp   r  J
t ;t  r4
W Pt E it i 2 Rt E 2t I 2
current H =     i.e., in absence of radiation looses, the time in
J J J J RJ
H  mst which fuse will melt is also independent on length
and varies with radius as r4.

| CURRENT ELECTRTICITY | 163


ACTIVE SITE EDUTECH-9844532971

SrSR||1212THTHCLASS
CLASS|NEET
|NEETPHYSICS
PHYSICS: VOL-1|
: VOL-1|
Illustration -18: Bulbs connected in Series:
A fuse wire with radius of 0.2mm blows Ø If bulbs (or electrical appliances) are connected
off with a current of 5 Amp. The fuse wire in series, the current through each resistance is
of same material, but of radius 0.3mm will same. Then power of the electrical appliance
blow off with a current of
P  R & V  R  P  i2Rt 
3 5 3
1) 5q Amp 2) Amp i.e. In series combination , the potential
2 2 difference and power consumed will be more
27 in larger resistance.
3) 5 Amp 4) 5 Amp
8 Ø When the appliances of power P1, P2 , P3 .... are
Solution: in series, the effective power consumed (P) is
3 3
i1 ž r1 ¬­ 2  0.2 ¬ 2 1 1 1 1
2 3; ž ­  žž ­­­     ......... i.e. effective power is
i rr i2 žŸ r2 ­®­ žŸ 0.3 ® P P1 P2 P3
less than the power of individual appliance.
27 Ø If ‘n’ appliances, each of equal resistance ‘R’
i2  5 Amp
8 are connected in series with a voltage source
Illustration -19: V2
‘V’, the power dissipated ‘ Ps ’ will be Ps  .
A 1 kW heater is meant to operate at 200 nR
V. (a) What is its resistance ? (b) How Bulbs connected in parallel:
much power will it consume , if the line Ø If bulbs (or electrical appliances) are connected
voltage drops to 100 V ? (c) How many in parallel, the potential difference across each
units of electrical energy will it consume 1 1
resistance is same. Then P  and I  .
in a month (of 30 days) if it operates 10 hr R R
i.e. The current and power consumed will be
daily at the specified voltage ?
more in smaller resistance.
Solution : Ø When the appliances of power P1, P2 , P3 .... are
a) The resistance of an electric appliance is given by in parallel, the effective power consumed(P) is
200   40 P  P1  P2  P3  .........
2 2
V
, R  S so, R  i.e. the effective power of various electrical
W 1000
appliance is more than the power of individual
b) The ‘ actual power ‘ consumed by an electric appliance.
appliance is given by , Ø If ‘n’ appliances, each of resistance ‘R’ are
2 connected in parallel with a voltage source ’V’,
V  the power dissipated ‘Pp’ will be
P   A  W
 VS  V2 nV 2
Pp  
2 R / n R
so, P   100  1000  250W
  Pp
 200   n 2 or  Pp  n 2 PS
c) The total electrical energy consumed by an PS
electric appliance in a specified time is given This shows that power consumed by ‘n’ equal
by, resistances in parallel is n2 times that of power
W1h1 consumed in series , if voltage remains same.
E kWh Ø In parallel grouping of bulbs across a given
1000 source of voltage, the bulb of greater wattage
will give more brightness and will allow more
so, E  1000  (10  30)  300kWh current through it, but will have lesser resistance
1000
and same potential difference across it.

164 | CURRENT ELECTRTICITY |


ACTIVE SITE EDUTECH-9844532971

Sr | 12TH CLASS |NEET PHYSICS : VOL-1|


Ø For a given voltage V, if resistance is changed V2 V2
then P1  and P2 
R R1 R2
from ‘R’ to  n  , power consumed changes
 
P1 R2 R1 P2 500
    5
from ‘P’ to ‘nP’; P’ 
V2 R
where R  , then P2 R1 (or) R 2 P1 100
R’ n
If the two bulbs are in series with the mains,
V2 nV 2 the same current ‘i’ flows through each of them.
P    nP .
R / n R Let P1 and P2 be the powers dissipated by
Ø If t1, t 2 are the time taken by two different coils two bulbs, then
for producing same heat with same supply, then P11  i 2 R1 and P21  i2 R 2
(i) If they are connected in series to produce P11 R1
same heat, time taken t  t1  t 2   5 or P11  5P21
P21 R2
(ii) If they are connected in parallel to produce
Since 100 watt bulb dissipates more power, it
tt
same heat, time taken is t 12 . glows brighter
t1  t 2 Illustration -23:
Illustration -20: A cell develops the same power across two
A lamp of 100W works at 220 volts. What resistances R1 and R2 separately. The in-
is its resistance and current capacity ? ternal resistance of the cell is
Solution : Solution :
Power of the lamp, P = 100W Let r and E are the internal resistance and emf
Operating voltage, V = 220V of the cell. When connected across the resis-
Current capacity of the lamp, tance R1 in the circuit, current passing through
P 100 the resistance is
i   0.455A 2
V 220 E  E ­¬
i = P1  i R1  žž
2
­ R
Resistance of the lamp, R  V  (220)  484
2 2 R1 r ; žŸ R r ­­® 1
1
P 100 2
Illustration -21:  E ­¬
A 100W – 220V bulb is connected to 110V Similarly P2  žžž R r ­­­ R 2 ; Given that P1 = P2
Ÿ 2 ®
source. Calculate the power consumed by Substituting the values, we get;
the bulb.
Solution : r  R1R 2
Power of the bulb, P = 100W Illustration -24:
Operating voltage, V = 200V A 100 W bulb B1 and two 60 W bulbs B2
Resistance of the bulb,
and B3, are connected to a 250V source,
V 2 (220)2
R   484 as shown in the figure. Now W1 , W2 and
P 100 W3 are the output powers of the bulbs B1,
Actual operating voltage, V1 = 110 V
Therefore, power consumed by the bulb, B2 and B3 respectively. Then find the ra-
(V1 )2 (110)2
tio of output powers of the bulbs .
P1    25W.
R 484
Illustration -22:
A 100W and a 500W bulbs are joined in
series and connected to the mains. Which
bulb will glow brighter ?
Solution :
Let R1 and R2 be the resistances of the two
bulbs. If each bulb is connected separately to
the mains of voltage V,

| CURRENT ELECTRTICITY | 165


ACTIVE SITE EDUTECH-9844532971

SrSR||1212THTHCLASS
CLASS|NEET
|NEETPHYSICS
PHYSICS: VOL-1|
: VOL-1|
So, the power dissipated in an external
Solution :
resistance is maximum if that resistance is equal
V2 to internal resistance of the source supplying
Resistance of a bulb is R  where P de- the current to that device.
P
notes the power of the bulb. Illustration -25:
= Resistance of B1(R1) = V2 / 100 A battery of internal resistance 4 is
Resistance of B2 (R2) = V2 / 60 connected to the network of resistances as
Resistance of B3(R3) = V2 / 60 shown. What must be the value of R so
= I1 = Current in that maximum power is delivered to the
250 250q300 network ? Find the maximum power ?
B1  
R1 R 2
8V2
I2 = Current in
250 250q300
B2  
R1 R 2
8V2
I3 = Current in B3 = I1 as B1, B2 are in series
= output power of B1 is W1 = I12 R1 Solution :
i) According to maximum power transfer theorem
 250q300 ­¬2 V 2
= W1  žž q R ext  R int
žŸ 8V 2 ­­® 100
3R q6R 4
2 2   4 º R   28
 250 q300 ¬­ V 9R 2
W2  I 22 R 2 or W2  žž ­ q
Ÿž 8V 2 ®­ 60 ii) Pmax  i2 R ext
 250 q300 ¬­2 V 2  E ¬­2 E2
W3  I 22 R 3 or W3  žž q
žŸ 8V 2 ­­® 60 ž

žŸ 4 4 ­­®
q4 
16
= W1 : W2 : W3  15 : 25 : 64 or W1 < W2 < W3 Consumption of Electrical Energy:
Ø Units of electrical energy consumed by an
Maximum power transfer theorem
electrical appliance =
Number of watts× Number of hours
1000
It is in KWH.
CELLS
Consider a device of resistance R connected
to a source of e.m.f E and internal resistance r Ø Primary Cells: Voltaic, Leclanche, Daniel
and Dry cells are primary cells. They convert
 E 
as shown. Current in the circuit is i   . chemical energy into electrical energy. They
R  r  can’t be recharged. They supply small currents.
Power dissipated in the device is P = i2R Secondary Cells (or) Storage Cells:
E R 2 Ø Electrical energy is first converted into chemical
P energy and then the stored chemical energy is
(R  r)2 converted into electrical energy due to these
For maximum power dissipated in the device cells.
Ø These cells can be recharged.
dP d  E2 R 
0   0 Ø The internal resistance of a secondary cell is
dR dR  (R  r)2  low where as the internal resistance of a primary
On simplification, we can get R = r cell is large.
166 | CURRENT ELECTRTICITY |
ACTIVE SITE EDUTECH-9844532971

Sr | 12TH CLASS |NEET PHYSICS : VOL-1|

emf of a Cell: The energy supplied by the


battery to drive unit charge around the circuit
is defined as electro motive force of the cell.
Ø EMF is also defined as the absolute potential
V
difference between the terminals of a source, i ........... (1)
R
when no energy is drawn from it. i.e., in the
E
open circuit of the cell. It depends on the nature and also i  ........... (2)
of electrolyte used in the cell. R r

iR + ir = E, V +ir = E, V = E – ir
Unit :J/C (or) Volt Lost volts: It is the difference between emf
Ø emf of a cell depends on and P.D. of a cell. It is used in driving the current
a) metal of electrodes between terminals of the cell.
b) nature of electrolyte Lost volts E - V = i r
Note: Formulae related with cells
c) temperature
E V ..................(I)
Ø emf of the cell is independent of i
r
a) area of plate
E V ....................(II)
b) quantity of electrolyte r
c) distance between plates i
 E V 
d) size of the cell r     E  V  R   E  1 R
Internal Resistance of a Cell  V   V  V 
.... (III)
 R 
Ø It is the resistance offered by the electrolyte of ER
the cell. Ø V = iR =
It depends on
R  r 
V R
1 Ø Fractional energy useful = 
i) area of the electrodes ( r ) E Rr
A Ø % of fractional useful energy
ii) nature of electrolyte , concentration( r C ) V   R 
iii) area of cross section of the electrolyte =   100 =  100
through which the current flows and E Rr
iv) age of the cell. V’ r
Ø Internal resistance of an ideal cell is zero. Ø Fractional energy lost, 
E Rr
Terminal Voltage:  ’
When no current flows through the cell, the Ø  V 100   r 100
% of lost energy,  E 
circuit is said to be an open circuit. This is shown   Rr
in figure. E V 
Ø internal resistance, r =  V  R
 

DIFFERENT CONCEPTS WITH A CELL


Ø When the cell is charging, the EMF is less than
the terminal voltage (E < V) and the direction
In such a case, the potential difference (p.d) of current inside the cell is from + ve terminal
across the terminals of the cell, called the to the –ve terminal.
terminal voltage (V) will be equal to the emf
(E) of the cell.
If an external resistance R is connected across
the two terminals of the cell, as in figure then V=E+ir
current flows in the closed circuit.,

| CURRENT ELECTRTICITY | 167


ACTIVE SITE EDUTECH-9844532971

SrSR||1212THTHCLASS
CLASS|NEET
|NEETPHYSICS
PHYSICS: VOL-1|
: VOL-1|
Ø When the cell is discharging, the EMF is greater through a resistor. When it is connected
than the terminal voltage (E >V) and the to another resistor of resistance of 10  in
direction of current inside the cell is from – ve
terminal to the +ve terminal. parallel, the current is 6A. Find the
resistance of the first resistor.
Solution:

V = E – i r ; Hence E  V
Ø Power delivered will be maximum when R = r.
E2
So Pmax 
4r
Ø This statement in generalized form is called
‘maximum power transfer theorem’ Current through R1 in the first case i1 = 5A
Current in the second case i2 = 6A
Effective resistance in the second case
R1R2 RR
1 2
R ; V  I1R1 and V  I2 R  R
R1  R2 1 2

R1R2 R2
I1R1  I2  I1  I2
Here the % of energy lost and energy useful R1  R 2 R1  R 2
are each equal to 50% 10
5  6  5(R1  10)  60
Back EMF: When current flows through the R1  10
electrolyte solution, electrolysis takes place with 5R1 + 50 = 60, 5R1 = 10
a layer of hydrogen and this hinders the flow of
10
current. In the neighborhood of both electrodes, R1   2
5
the concentrations of ions get altered. This
opposing EMF is called back EMF and the Illustration -28:
phenomenon is called Electrolytic polarisation. When a battery is connected to the
To reduce back emf manganese dioxide (or) resistance of 10 , the current in the circuit
potassium dichromate is added to electrolyte is 0.12A. The same battery gives 0.07A
of cell. current with 20 . Calculate the e.m.f. and
Illustration -26: internal resistance of the battery.
When a current drawn from a battery is Solution :
0.5A, its terminal potential difference is We know that E  Ir  IR
20V and when current drawn from it is I2 R 2  I1R1
2.0A, the terminal voltage reduces to 16 V. I1r  I1R1  I2 r  I2 R2 ; r 
I1  I2
Find out the e.m.f and internal resistance
of the battery. 0.07  20  0.12  10 1.4  1.2 0.2
r    4
0.12  0.07 0.05 0.05
Solution:
Internal resistance r  4
We know e. m. f E  Ir  IR
V = E --- Ir ; I = 0.5 A, V = 20 Volt, we have 0.12  4  0.12  10  0.48  1.2 ; E = 1.68 volt.
20 = E – 0.5r ....... (i)
I = 2 A, V = 16 Volt, we have GROUPING OF CELLS
16 = E – 2r ....... (ii) Electric Cells in Series: When ‘n’
By solving (i) and (ii) identical cells each of EMF ‘E’ and internal
resistance ‘r’ are connected in series to an
we get E = 21.3V, r = 2.675  . external resistance ‘R’, then
Illustration -27: Ø total emf of the combination = n E
An ideal battery passes a current of 5A Ø effective internal resistance = n r

168 | CURRENT ELECTRTICITY |


ACTIVE SITE EDUTECH-9844532971

Sr | 12TH CLASS |NEET PHYSICS : VOL-1|


Ø Current through external resistance Illustration -29:
nE Two cells A and B with same e.m.f of 2 V each
i= and with internal resistances
R  nr
rA  3.5 and rB  0.5 are connected in
 series with an external resistance R  3 .
Ø If R << n r then i = = current from one cell
r Find the terminal voltages across the two
nE cells.
Ø If R>> n r then i = Solution:
R
Ø If two cells of different emf’s are in series Current through the circuit
 22 4
E1  E2 i  
Eeq = E1 + E2 ; req = r1 + r2 ; i  r  r  R  R  r  3  3.5  0.5  7
1 2
i) R  3, rA  3.5 , E  2V
Terminal voltages A, VA  E  irA
4
 2   3.5  0 volt
7
ii) rB  0.5 8, R 3 8, E  2V
Terminal voltage at B, VB  E  irB
Terminal voltage across the first cell V1=E1- ir1 4
 2   0.5  1.714 volts.
Terminal voltage across the second cell V2=E2-ir2 7
Ø If one of the cell is in reverse connection Illustration -30:
( E1 > E2 ) then Eeq = E1 - E2 Two cells A and B each of 2 V are
connected in series to an external
E1  E2 resistance R=1 ohm . The internal
req = r1 + r2 ; i  r  r  R resistance of A is rA =1.9 ohm and B is rB
1 2
=0.9 ohm. Find the potential difference
between the terminals of A.
Solution:
Total current through the circuit
net emf
i
Total resist an ce

First cell is discharging then V1 = E1 - ir1


Second cell is charging then V2 = E2 + ir2
cell having less emf in charging state.
WRONGLY CONNECTED CELLS 4 4
  A
Ø By mistake if ‘m’ cells out of ‘n’ cells are 1  1.9  0.9 3.8 .
wrongly connected to the external resistance potential difference at A, VA    ir ,
‘R’
4
(a) total emf of the combination = (n – 2m) E  2  1.9 = 2 – 2 = 0.
3.8
(b) total internal resistance = n r Illustration -31:
(c) total resistance = R + n r In the given circuit as shown below,
n  2m E calculate the magnitude and direction of
(d) current through the circuit (i) =
R  nr the current

| CURRENT ELECTRTICITY | 169


ACTIVE SITE EDUTECH-9844532971

SrSR||1212THTHCLASS
CLASS|NEET
|NEETPHYSICS
PHYSICS: VOL-1|
: VOL-1|

r E
Ø If R >> , then i = = current from one cell.
n R
r nE
Ø If R << , then i =
n r
Ø If two cells of emf E1 and E2 having internal
Solution: resistances r1 and r2 are connected in parallel
Effective resistance of the circuit is to an external resistance ‘R’, then
Reff  2  2  1  5
V V
 total current in the circuit is i  1 2
Reff
10  5
i=  1A
5
Since the cell of larger emf decides the direction
of flow of current, the direction of current in E 1 r 2  E 2r 1
the circuit is from A to B through e the effective emf, E = r 1  r2
Illustration -32:
r1 r2
A voltmeter of resistance 500 8 is used to the effective internal resistance, reff  r  r
measure the emf of a cell of internal resis- 1 2

tance 48 . The percentage error in the E


reading of the voltmeter will be Current through the circuit, i  r  R
eff
Solution: i = i1 + i2
V = E – ir
E1  i R E2  iR
= Percentage error 
%E ir
q100  q100 i1 = and i 2  r2
r1
E E
Potential difference across R, i.e terminal
 E ¬­
žž r
žŸ R r ­­®  r ¬­
ER
 q100  žž ­q100 potential of the cells is V  iR  R  r
E Ÿž R r ®­ eff

Ø When the cell E2 is reversed in polarity then


 4 ­¬
 žž q100  0.8%
žŸ 500 4 ­­® we should use - E2 in all the above equations.
Mixed Grouping: If n identical cells are
ELECTRIC CELLS IN PARALLEL connected in a row and such m rows are
When ‘n’ identical cells each of emf ‘E’ and connected in parallel then
internal resistance ‘r’ are connected in parallel
to an external resistance ‘R’, then
Ø total emf of the combination = E
r
Ø effective internal resistance =
n
r
Ø total resistance in the circuit = R +
n
Ø current through the external resistance
E nE
i=  Ø Equivalent emf of the combination Ee q = nE
r nRr
R
n

170 | CURRENT ELECTRTICITY |


ACTIVE SITE EDUTECH-9844532971

Sr | 12TH CLASS |NEET PHYSICS : VOL-1|


Ø Equivalent internal resistance of the
Distribution of current at a junction in the
nr
combination req = circuit
m
Ø Main current flowing through the load I1 + I2 = I3 + I4 or I1 + I2 – I3 – I4 = 0
nE nmE If we take currents approaching point A in
i 
nr mR  nr figure as positive and that leaving the point as
R
m negative, then the above relation may be written
nr as
Ø Condition for maximum power R  and
m I1 + I2 +(–I3) + (–I4) = 0
E2 I  0
Pmax  mn 
4r Note: Kirchhoff’s first law is accordance with law
Ø Condition for maximum current of conservation of charge, since no charge
R r can accumulate at a junction.
+ =minimum
n m
(ii) Kirchoff`s Second Law (Loop Law or
d  mR r   N
 0;  n  m  Potential law): Kirchhoff’s second law
dm  N m 
states that the algebraic sum of the changes
R r R r
 2  0; i.e.,  (N = n x m) in potential around any closed loop is zero.
N m n m
So in case of mixed grouping of cells, (Kirchhoff’s second law) can be expressed as
current in the circuit will be maximum 4V  0 .
R r 
when    In terms of potential drops and emfs, the law
 n m is expressed as 4 iR
4E  0
nE mE
I max   Sign conventions:
2 R 2r
Ø Total number of cells = m  n (a) The change in potential in traversing a resistance
KIRCHHOFF’S LAWS in the direction of current is –IR while in the
When the circuit is complicated to find current opposite direction +IR as shown in the figure.
Kirchhoff’s laws are formulated.
(i) Kirchhoff`s First Law (Junction Law or
Current law) : It states that the sum of the
currents flowing into a junction is equal to the VA - IR = VB VB + IR = VA
sum of the currents flowing out of the junction. VA - VB = IR VA - VB = IR
Or (b) The change in potential in traversing an emf
“The algebraic sum of currents at a junction is
zero”. source from negative to positive is +E while in
the opposite direction –E , irrespective of the
direction of current in the circuit as shown in
the figure.

VA - E = VB VB + E = VA
V A - VB = E VA - VB = E

| CURRENT ELECTRTICITY | 171


ACTIVE SITE EDUTECH-9844532971

SrSR||1212THTHCLASS
CLASS|NEET
|NEETPHYSICS
PHYSICS: VOL-1|
: VOL-1|

Example 1:
E1  iR  i1r1  0 or i1  E1  iR ..... (1)
r1 r1
E2 iR
E2  iR  i2 r2  0 ; i2   ..... (2)
r2 r2
Adding Eqs. (1), (2) we get
Apply the Kirchhoff’s second law to the loop i1  i2  ( E1 / r1 )  ( E2 / r2 )  iR(1/ r1  1/ r2 )
ABCDA, then or i[1  R(1/ r1  1/ r2 )]  ( E1 / r1 )  ( E2 / r2 )
– iR1 – iR2 – iR3 + E = 0 ;
( E / r )  ( E2 / r2 )
E i  1 1
=i 1  R (1/ r1  1/ r2 )
R1 R 2 R3

Illustration -33:
Example 2:
Find the emf (V) and internal resistance
(r) of a single battery which is equivalent
to a parallel combination of two batteries
of emfs V1 and V2 and internal resistances
r1 and r2 respectively, with polarities as
Apply the kirchhoff’s second law to the loop
shown in figure
ADCBA, then
–iR – ir2 + E2 – E1 – ir1 = 0
E 2  E1
i(r1 + R + r2) = E2 – E1 º i 
r1 r2 R
Note:
Solution:
1) This law represents “conservation of energy”
emf of battery is equal to potential difference
2) If there are n meshes in a circuit, the number of
across the terminals, when no current is drawn
independent equations in accordance with loop from battery (for external circuit) [Here, all the
rule will be (n – 1). elements in the circuit are in series]
Application : This is the most general case Current in internal circuit = i
of parallel grouping in which E and r of Net emf V V2
different cells are different and the positive =i or i= 1
Total resistance r1 r2
terminals cells are connected as shown

= VA  VB  V1  ir1
[ ' V1 cell is discharging]
Kirchhoff’s second law in different loops gives
the following equations,  V V2 ¬­
or VA  VB  V1 žž 1 ­r
žŸ r1 r2 ®­­ 1

172 | CURRENT ELECTRTICITY |


ACTIVE SITE EDUTECH-9844532971

Sr | 12TH CLASS |NEET PHYSICS : VOL-1|


Illustration -35:
V1r2  V2 r1 Solve for current values in figure.
or VA  VB  r r
1 2
= Equivalent emf of the battery = V
V1r2  V2 r1
= V  r r
1 2
(ii) Internal resistance of equivalent battery. r1 and
r2 are in parallel. Solution :
1 1 1 r1r2 Applying Kirchhoff’s first law at the junction B
 or r 
r r1 r2 r1 r2 we have i1 + i2 = i3 ....... (1)
Illustration -34: Applying Kirchhoff’s second law to loop
ABEFA
In the given circuit values are as follows
– 12 + i2 x 1.5 – i1 x 1 + 8 = 0
E1  2V , E2  4V , R1  1  and R2  R3  1  .
i1 – 1.5 i2 = – 4. ....... (2)
Calculate the Currents through R1, R2 and
R 3. From loop BCDEB
– (i2 x 1.5) – (i3 x 9) + 12 = 0
1.5 i2 + 9i3 = 12 ....... (3)
on solving i1 = – 1A and i3 = 1 A
WHEATSTONE’S BRIDGE

Solution:

Let i1 , i2 are currents through R1 and R3.


(i1 + i2) is current through R2.
Their direction are taken as shown Condition for balancing of bridge :
Applying Kirchhoff`s first law at junction B and
From Kirchhoff’s second law for AGFBA loop
D we get I1 – I3 – IG = 0 ; and I2 + IG – I4 = 0
i1 R1  (i1 i2 ) R2 E1  0 ; i1  i1  i2  2
Applying Kirchhoff’s second law for closed
2i1  i2  2  (1)
loop ABDA, I1P  IGG I2 R  0
From Kirchhoff’s second law for BCDEB loop Applying Kirchhoff’s second law for closed
i2 R3  (i1 i2 ) R2 E2  0 ; loop BCDB , I3Q I4S IGG  0
i2  i1  i2  4 The values of P, Q, R, S are adjusted such that
i1  2i2  4  (2) I G becomes zero. At this stage the bridge is
Solving equation (1) and (2) we get i1 = 0A, i2 set to be in balance condition.
= 2A i.e., In balanced condition of bridge IG = 0
Thus currents through R1 is 0, while through R3  In balanced condition the above equations
and R2 are 2A each. respectively become

| CURRENT ELECTRTICITY | 173


ACTIVE SITE EDUTECH-9844532971

SrSR||1212THTHCLASS
CLASS|NEET
|NEETPHYSICS
PHYSICS: VOL-1|
: VOL-1|
I1 = I3 ........(1)
VAB  VA  VB  i1P  i
R  S  P
and I2 = I4 ........(2) PQ RS
I1P  I 2 R ...........(3)
VAD  VA  VD  i2 R  i
P  Q R
I 3Q  I 4 S ............(4)
PQ RS
Dividing equation (3) by equation (4)
I1P I 2 R i
 VB  VD  P  Q  R  S P  Q  R   R  S  P 
I 3Q I 4S
Using equations (1) and (2) we get i
P R

PQ RS
QR  PS 
 .......(5)
Q S
if QR  PS , VB  VD  current flows from B to
This is the balancing condition for Wheat stone
bridge. D
APPLICATIONS ON WHEATSTONE BRIDGE QR  PS , VB  VD  current flows from D to B

1. We can compare two unknown resistances R QR  PS , VB  VD  Balanced bridge


P R Illustration -36:
and S from  Determine the current in each branch of
Q S
2. In place of resistances we can use capacitors the network shown in fig.
to form a D.C. Wheatstone bridge with four
capacitors of capacitances C1, C2, C3 and C4.
C1 C3
The balancing condition will be C  C
2 4
3. It has been found that the bridge has the
greatest sensitivity when the resistances are as
nearly equal as possible.
The bridge is most sensitive if P = Q = R = S.
4. Equivalent resistance of balanced bridge across
the ends of battery when the bridge is balanced
is given by

Rtotal 
P  Q R  S 
P Q  R  S
Application : Direction of current in an
unbalanced wheatstone’s bridge :
Solution:
Apply KVL in loop ABDA
10I1 5 I  2I1
5 I  I1
 0
2I  5I a .......(1)
Apply KVL in ADCEFA loop
5 I  I1
10I1 10 10I=0
5I1 + 15I = 10 .......(2)

174 | CURRENT ELECTRTICITY |


ACTIVE SITE EDUTECH-9844532971

Sr | 12TH CLASS |NEET PHYSICS : VOL-1|


From equation (1) and (2)
Illustration -37:
10 21 4 In the experimental arrangement of the
I= ;
1
I=  A
17 5 17 of the meter bridge as shown , if AC
4 corresponding to null deflection of
Current in AB branch  galvanometer is x, what would be its value
17 if the radius of the wire AB is doubled?
10 4 6
=I  I1    A
17 17 17
Current in DB branch
10 8 2
I  2Ia = - = A
17 17 17
Ø Metre bridge: It works on the principle
Solution:
P R For null deflection of galvanometer in a meter
of Wheatstone Bridge    bridge experiment,
Q S 
R1 R AC R x
 or 1 
R 2 R CB R 2 100  x

Since R1 / R2 remains constant, x / (100-x)


also remains constant. The value of x remains
as such.
= Length of AC = x
Illustration -38:
A resistance of 2 is connected across
Ø When the Meter bridge is balanced then one gap of a meter bridge (the length of
R l1 l1 the wire is 100 cm) and an unknown
  resistance, greater than 2 , is connected
X l2 100  l1
across the other gap. When these
Where l1 is the balancing length from the left resistances are interchanged, the balance
end. point shifts by 20 cm. Neglecting any
Note: corrections, the unknown resistance is
1. If resistance in the left gap increases or Solution:
resistance in the right gap decreases, balancing Refer to the diagram Apply the conditions of
point shifts towards right side. the balanced Wheatstone’s bridge for the two
2. If resistance in the left gap decreases or cases.
resistance in the right gap increases, balancing
point shifts towards left.
3. If a cm, b cm are the end corrections at A and
R l1  a
B, then X  l  b
2
2 A
4. Meter bridge is more sensitive when l1 = 50  ..................................(i)
cm x 100  A
5. The resistance of copper strip is called end x A 20
 ....................................(ii)
resistance 2 80  A
Equations (i) and (ii) give x  38

| CURRENT ELECTRTICITY | 175


ACTIVE SITE EDUTECH-9844532971

SrSR||1212THTHCLASS
CLASS|NEET
|NEETPHYSICS
PHYSICS: VOL-1|
: VOL-1|
Potentiometer: Potentiometer is an R
instrument which can measure accurately the If the resistance per unit length of the wire ,
L
emf of a source or the potential difference is denoted by  , the potential drop across the
across any part of an electric circuit without wire is
drawing any current. V  I   A
Principle : The principle of potentiometer states
V
that when a constant current is passed through is called potential drop per unit length of
a wire of uniform area of cross-section, the l
the potentiometer wire or potential gradient of
potential drop across any portion of the wire is the wire. It is given by
directly proportional to the length of that portion.
The principle of potentiometer require that V  E R
 I   
i) potentiometer wire should be of uniform l  r  R  RS  L
area of cross-section and Thus, the unknown voltage V is measured when
ii) current through the wire should remain no current is drawn from it.
constant. 1) When specific resistance (S) of potentiometer
Theory of potentiometer : The end of the wire is given then potential gradient
potentiometer wire AB are connected to a
standard cell of emf E or a source of emf E that IS IS
X 
supplies constant current. The current through A Q r2
the potentiometer wire can be varied by means where A = area of cross - section of potentiometer
of a series resistance Rs which is adjustable. wire r = Radius of potentiometer wire.
2) When two wires of length L1 and L2 and
resistance R1 and R2 are joined together to
x1 R1 L2

form the potentiometer wire, then
x2 L1 R2
Potential gradient depends on
a) Resistance per unit length of the
potentiometer wire (  =R/L)
b) Radius of cross-section of the potentiometer
Let r be the internal resistance of the cell of
wire, when the series resistance is included
emf E connected across the potentiometer wire
in the circuit and cell in the primary circuit
of length L and resistance R. The current
is not ideal.
through the potentiometer wire is
c) Current flowing through potentiometer wire.
E
I d) emf of the cell in primary circuit
r  R  Rs
e) Series resistance in the primary circuit
The potential of the wire decreases from the
end A to the end B. The potential fall or potential f) Total length (L) and resistance (R) of the
drop across a length l of the potentiometer potentiometer wire.
wire is g) If cell in primary circuit is ideal and in the
V = Current x Resistance of length l of the absence of series resistance potential
gradient only depends on emf of cell in
R
potentiometer wire  I    l primary circuit and length of potentiometer
L wire

176 | CURRENT ELECTRTICITY |


ACTIVE SITE EDUTECH-9844532971

Sr | 12TH CLASS |NEET PHYSICS : VOL-1|


TO DETERMINE THE INTERNAL
RESISTANCE OF A PRIMARY CELL: Ø Let E1  E2 and both are connected in series.
If balancing length is l1 when cells assist each
other and it is l2 when they oppose each other
as shown then:

E1  E2   xl1 E1  E2   xl2


E1  E2 l1 E1 l1  l2
  (or) E  l  l
Ø Initially in secondary circuit key K’ remains E1  E2 l2 2 1 2
Comparison of resistances:
open and balancing length l1  is obtained.
Let the balancing length for resistance R1 (when
Since cell E is in open circuit so it’s emf balances
XY is connected) be l1 and let balancing length
on length l1
for resistance R1  R2 (when YZ is connected)
i.e E  xl1 .....(i)
Ø Now key K ’ is closed so cell E comes in be l2 . keeping X constant
closed circuit. If the process of balancing is
repeated again keeping constant then potential
difference V balances on length l2
i.e V  xl2 .....(ii)
Ø By using the formula for internal resistance
E 
r    1  .R /
V 
Where E = emf of cell in secondary circuit
V = Terminal voltage
 l1  l2  Then iR1  xl1 and
i.e p.d on R, r    R’ R2 l2  l1
 l2  i  R1  R2   xl2  
R1 l1
E l E l
'  1 , 1  1 1 To determine thermo emf:
V l2 V l2
COMPARISON OF EMF’S OF TWO CELLS
Ø Let l1 and l 2 be the balancing length with the
cell E1 and E2 respectively, then E1  xl1 and
E1 l1
E2  xl2  
E2 l2
Ø The value of thermo-emf in a thermocouple for
ordinary temperature difference is very low
10 6

volt . For this the potential gradient x
must be also very low 10 4 V / m  . Hence a
high resistance (R) is connected in series with
the potentiometer wire in order to reduce
current in the primary circuit

| CURRENT ELECTRTICITY | 177


ACTIVE SITE EDUTECH-9844532971

SrSR||1212THTHCLASS
CLASS|NEET
|NEETPHYSICS
PHYSICS: VOL-1|
: VOL-1|
Ø The potential difference across R must be equal Illustration -40:
to the emf of standard cell A cell of e.m.f 2 volt and internal
E resistance 1.5 is connected to the ends
i.e iR  E0 i  0
R of 1m long wire. The resistance of wire is
Ø The small thermo emf produced in the
thermocouple e  xl 0.5 / m . Find the value of potential
gradient on the wire.
iR| iR ’l Solution:
Ø x  i   e
L L
where L = Length of potentiometer wire, I R  E  R 2  0.5
X    =0. 5 V/m
 =resistance per unit length, l  balancing L  R  r  L 0.5  1.5
length of e and R| = Resistance of Illustration -41:
potentiometer wire In a potentiometer experiment the
SENSITIVITY OF POTENTIO METER balancing length with a cell is 560 cm.
When an external resistance of 10  is
1. Sensitivity of potentio meter is estimated by its connected in parallel to the cell, the
potential gradient. balancing length changes by 60 cm. Find
2. Sensitivity is inversely proportional to potential the internal resistance of the cell.
gradient so lower the potential gradient higher Solution:
will be the sensitivity. Balancing length A1  560 cm
Change in balancing length (A1  A 2 )  60 cm
3. The best instrument for accurate measurement 560  A 2  60
of e.m.f. of a cell is potentiometer, because it  A 2  500 cm
does not draw any current from the cell.
 A A ¬ 60 6
rR žžž 1 2 ­­­ º r10q  1.2 8 .
Illustration -39: Ÿ A 2 ®­ 500 5

The length of a potentiometer wire is 1m Illustration -42:


In a potentiometer experiment, when a
and its resistance is 4 . A current of 5
battery of e.m.f. 2V is included in the
mA is flowing in it. An unknown source
secondary circuit, the balance point is
of e.m.f is balanced on 40 cm length of this
500cm. Find the balancing length of the
wire, then find the e.m.f of the source. same end when a cadmium cell of e.m.f.
Solution: 1.018V is connected to the secondary
circuit.
R 5 4
x  I  I   20 mV 1 E
1 A
L 1 Solution: E  A E  A
2 2
E = l x = 0. 40 x 20 = 8 mV E2 1.018
A2   A1   500  254.5cm .
E1 2

178 | CURRENT ELECTRTICITY |


ACTIVE SITE EDUTECH-9844532971

Sr | 12TH CLASS |NEET PHYSICS : VOL-1|


JG
CLASS ROOM EXERCISE 5. When electric field ( E ) is applied on the
ends of a conductor, the free electrons
TEACHING QUESTIONS starts moving in direction
JG JG
1) similar to E 2) Opposite to E
ELECTRIC CURRENT AND DRIFT JG
3) Perpendicular to E 4) Cannot be predicted
VELOCITY
6. The drift speed of an electron in a metal is
1. If n, e,  , m, are representing electron of the order of
density, charge, relaxation time and mass 1) 10–13 m/s 2) 10–3 mm/s
of an electron respectively then the 3) 10–4 m/s 4) 10–30 m/s
resistance of wire of length l and cross 7. In metals and vacuum tubes charge
sectional area A is given by carriers are
1) electrons 2) protons
ml 2mA 3) both 4) positrons
1) 2)
ne 2 $ ne 2  8. The electric intensity E, current density j
and conductivity  are related as :
ne2 A 1) j  E 2) j  E / 
3) ne 2 A 4)
2m 3) jE   4) j   2 E
2. Among the following dependences of drift 9. Electric field (E) and current density (J)
velocity vd on electric field E, Ohm’s Law have relation
obeyed is 1) E  J 1 2) E  J

1) vd E 2) vd E 2 1 1
3) E  4) E 
2
2
J J
3) vd  E 4) vd  constant 10. A current of 5A is passing through a
metallic wire of cross section area
3. A steady current is passing through a
linear conductor of nonuniform cross- 4  106 m 2 . If the density of the charge
section. The net quantity of charge carriers in the wire is 5  1036 / m3 , the drift
crossing any cross section per second is velocity of the electrons is
1) independent of area of cross-section 1 1
1) m / sec 2) m / sec
2) directly proportional to the length of the 16 32
conductor
1 1
3) directly proportional to the area of cross 3) m / sec 4) m / sec
64 128
section.
11. If the electron in a Hydrogen atom makes
4) inversely proportional to the area of the 6.25x1015 revolutions in one second, the
conductor current is
4. Given a current carrying wire of non- 1) 1.12 mA 2) 1 mA
uniform cross section. Which of the 3) 1.25 mA 4) 1.5 mA
following quantity or quantities are 12. The current through a wire connected to a
constant throughout the length of the wire? condenser varies with time as i  2t  1 A
1) current, electric field and drift speed The charge transport to the condenser
2) drift speed only from t  0 to t  5s is
3) current and drift speed 4) current only 1) 5C 2) 55C 3) 30C 4) 60C

| CURRENT ELECTRTICITY | 179


ACTIVE SITE EDUTECH-9844532971

SrSR||1212THTHCLASS
CLASS|NEET
|NEETPHYSICS
PHYSICS: VOL-1|
: VOL-1|
13. A copper wire of cross-sectional area 1) is constant throughout the wire
2.0 mm2 , resistivity = 1.7 108 m , 2) varies unpredictably
carries a current of 1 A. The electric field 3) decreases from P and Q
in the copper wire is 4) increases from P to Q
18. A current of 16 A is made to pass through
1) 8.5 105 V / m 2) 8.5 104 V / m
a conductor in which the number of density
3) 8.5 103 V / m 4) 8.5 102 V / m of free electrons is 4 × 1028 m–3 and its
area of cross section is 10–5 m 2. The
14. In a hydrogen tube it is observed that average drift velocity of free electrons in
through a given cross - section 3.13 x 1015 the conductor is (M-2012)
electrons per sec, moving from right to left
1) 1.6 × 10–4 ms–1 2) 2.5 × 10–4 ms–1
and 3.12 x 1015 protons per sec are mov-
ing from left to right. The electric current 3) 6.4 × 10–4 ms–1 4) 3.2 × 10–4 ms–1
in the discharge tube and its direction is 19. When 5V potential difference is applied
1) 1 mA towards left 2) 1 mA towards right across a wire of length 0.1m, the drift speed
3) 1.5 mA towards right 4) 2 mA towards left of electrons is 2.5 104 ms 1 . If the
15. An electron of mass m, moves around the electron density in the wire is 8 1028 m3 ,
nucleus in a circular orbit of radius ‘r’ the resistivity of the material is close to:
under the action of centripetal force ‘F’.
(Jee Mains 2015)
The equivalent electric current is
1) 1.6  108 m 2) 1.6 107 m
e F e Fr
1) 2) 3) 1.6  106 m 4) 1.6  105 m
2 mr  m

e Fm e Fm
3) 4)
2 r  r
16. The current in a conductor varies with time 1) 1 2) 1 3) 1 4) 4 5) 2 6) 3
‘t’ as I  3t  4t 2 . Where I in amp and t in 7) 1 8) 1 9) 2 10) 3 11) 2 12) 3
sec. The electric charge flows through the 13) 3 14) 2 15) 1 16) 3 17) 3 18) 2
section of the conductor between t = 1s and
t=3s 19) 4

100 127 140 150 OHM’S LAW AND FACTORS


1) C 2) C 3) C 4) C
3 4 3 3 EFFECTING RESISTANCE
17. A conductor has a non-uniform section as
shown in the figure. A steady current is 20. In an electric circuit containing a battery,
flowing through it. Then the drift speed of the charge (assumed positive) inside the
battery
the electrons (M-2012)
1) always goes form the positive terminal to
the negative terminal
2) may move from the positive terminal to
the negative terminal
3) always goes from the negative terminal to
the positive terminal
4) does not move .

180 | CURRENT ELECTRTICITY |


ACTIVE SITE EDUTECH-9844532971

Sr | 12TH CLASS |NEET PHYSICS : VOL-1|


21. From the following the quantity which is 29. With the increase of temperature, the ratio
analogous to temperature in electricity is of conductivity to resistivity of a metal
1) potential 2) resistance conductor
3) current 4) charge 1) Decreases 2) Remains same
22. The flow of the electric current through a 3) Increases 4) May increase or decrease
metallic conductor is 30. The conductivity of a super conductor, in
1) only due to electrons 2)only due to +ve the super conducting state is
charges 1) Zero 2) Infinity
3) due to both nuclei and electrons. 3) Depends on temp
4) can not be predicted. 4) Depends on free election
23. For making standard resistance, wire of 31. When a piece of aluminium wire of finite
following material is used length is drawn through a series of dies to
1) Nichrome 2) Copper reduce its diameter to half its original
3) Silver 4) manganin value, its resistance will become
24. Material used for heating coils is 1) Two times 2) Four times
1) Nichrome 2) Copper 3) Eight times 4) Sixteen times
32. Metals have
3) Silver 4) Manganin
1) Zero resistivity 2) High resistivity
25. A piece of silver and another of silicon are 3) Low resistivity 4) Infinite resistivity
heated from room temperature. The 33. Consider a rectangular slab of length L,
resistance of and area of cross-section A. A current I is
1) each of them increases passed through it, if the length is doubled
2) each of them decreases the potential drop across the end faces
3) Silver increases and Silicon decreases 1) Becomes half of the initial value
4) Silver decreases and Silicon increases 2) Becomes one-forth of the initial value
26. i-v graph for a metal at temperatures t1, t2, t3 3) Becomes double the initial value
are as shown. The highest temperature is 4) Remains Same
34. A metallic block has no potential difference
applied across it, then the mean velocity
of free electrons is (T = absolute
temperature of the block)
1) Proportional to T
2) Proportional to T 3) Zero
4) Finite but independent of temperature.
1) t1 2) t2 3) t3 4) t1  t2  t3
35. The resistance of a metal increases with
27. A certain piece of copper is to be shaped increasing temperature because
into a conductor of minimum resistance. Its 1) The collisions of the conducting electrons
length and cross sectional area should be with the electrons increases.
1) L and A 2) 2L and A/2 2) The collisions of the conducting electrons
3) L/2 and 2A 4) 3L and A/3 with the lattice consisting of the ions of the
metal increases
28. When light falls on semiconductors, their
3) The number of the conduction electrons
resistance decreases.
1) decreases 2) increases 4) The number of conduction electrons
3) does not change 4) can’t be predicted increase.

| CURRENT ELECTRTICITY | 181


ACTIVE SITE EDUTECH-9844532971

SrSR||1212THTHCLASS
CLASS|NEET
|NEETPHYSICS
PHYSICS: VOL-1|
: VOL-1|
36. In the absence of applied potential, the 41. The thermistors are usually made of
electric current flowing through a metallic 1) metals with low temperature coefficient of
wire is zero because resistivity
1) The average velocity of electron is zero 2) metals with high temperature coefficient of
2) The electrons are drifted in random direction resistivity.
with a speed of the order of 10 cm/s.
-2 3) metal oxides with high temperature
3) The electrons move in random direction with coefficient of resistivity
a speed of the order close to that of velocity 4) semiconducting materials having
of light. low temperature coefficient of resistivity
4) Electrons and ions move in opposite direction. 42. For a chosen non-zero value of voltage,
37. A long constantan wire is connected across there can be more than one value of
current in
the terminals of an ideal battery. If the wire
1) copper wire 2) thermistor
is cut in to two equal pieces and one of them
3) zener diode 4) manganin wire
is now connected to the same battery, what
43. At absolute zero silver wire behaves as
will be the mobility of free electrons now in
the wire compared to that in the first case? 1) Super conductor 2) Semi conductor
3) Perfect insulator 4) Semi insulator
1) same as that of previous value
44. Fuse wire is a wire of
2) double that of previous value
1) low melting point and low value of B
3) half that of previous value 2) high melting point and high value of B
4) four times that of previous value 3) high melting point and low value of B
38. Ohm’s law is not applicable for 4) low melting point and high value of B
1) insulators 2) semi conductors 45. A wire has resistance 12 . It is bent in
3) vacuum tube 4) all the above the form of a circle. The effective resistance
39. V - I graphs for two materials is shown in between two points across a diameter is
the figure. The graphs are drawn at two 1) 3 2) 6 3) 12  4) 24
different temperatures. 46. For what value of R the net resistance of
the circuit will be 18 ohms

1) 8 2) 10 3) 16 4) 24


1) T2  T1 r cot 2R 2) T1  T2 r sin 2R
47. The total current supplied to the circuit by
3) T2  T1 r tan 2R 4) T1  T2 r cos2R the battery is
40. Wires of Nichrome and Copper of equal
dimensions are connected in series in
electrical circuit. Then
1) More current will flow in copper wire
2) More current will flow in Nichrome wire
3) Copper wire will get heated more 1) 1A 2) 2A 3) 4A 4) 6A
4) Nichrome wire will get heated more

182 | CURRENT ELECTRTICITY |


ACTIVE SITE EDUTECH-9844532971

Sr | 12TH CLASS |NEET PHYSICS : VOL-1|


48. Using three wires of resistances 1 ohm, 54. Resistance of each 10  are connected as
2ohm and 3 ohm, then no.of different values shown in the fig. The effective resistance
of resistances that possible are between A and G is
1) 6 2) 4 3) 10 4) 8
49. Six resistances of each 12 ohm are
connected as shown in the fig. The
effective resistance between the
terminals A and B is
1) 16  2) 20  3) 12  4) 8 
55. Which arrangement of four identical
resistances should be used to draw
maximum energy from a cell of voltage V
1) 8  2) 6  3) 4  4) 12 
50. Current ‘i’ coming from the battery and
ammeter reading are

56. If four resistances are connected as shown


3 1 1 1
1) A, A 2) A, A in the fig. between A and B the effective
8 8 8 8 resistance is
2 1
3) 2 A, A 4) 2 A, A
3 8
51. In the circuit shown, the reading of the
voltmeter and the ammeter are

1) 4  2) 8  3) 2.4  4) 2 
57. A letter 'A' is constructed as a uniform wire
of resistance 1 ohm/cm. The sides of the
letter are 20 cm long and the cross piece
in the middle is 10cm long while the vertex
angle is 600. The resistance of the letter
1)4V, 0.2A 2)2V, 0.4A between the two ends of the legs is
3)3V, 0.6A 4)4V, 0.04A 1) 40/3  2) 80/3  3) 40  4) 10 
52. The resistance of a wire of 100 cm length 58. Find the value of colour coded resistance
is 10  . Now, it is cut into 10 equal parts shown is fig
and all of them are twisted to form a single
bundle. Its resistance is
1) 1  2) 0.5  3) 5  4) 0.1 
53. A metallic wire of resistance 20 ohm
stretched until its length is doubled. Its
resistance is
1) 520  10% 2) 5200  1%
1) 20  2) 40  3) 80  4) 60 
3) 52000  10% 4) 52000  1%
| CURRENT ELECTRTICITY | 183
ACTIVE SITE EDUTECH-9844532971

SrSR||1212THTHCLASS
CLASS|NEET
|NEETPHYSICS
PHYSICS: VOL-1|
: VOL-1|

59. The resistance of a wire is 2  . If it is 66. Two different wires have specific
drawn in such a way that it experiences a resistivities, lengths, area of cross-sections
longitudinal strain 200%. Its new are in the raio 3:4, 2:9 and 8:27. Then the
resistance is ratio of resistance of two wires is
1) 4  2) 8  3) 16  4) 18  16 9 8 27
1) 2) 3) 4)
60. ‘n’ conducting wires of same dimensions 9 16 27 8
but having resistivities 1, 2, 3,...n are 67. Two wires made of same material have
connected in series. The equivalent their length are in the ratio 1:2 and their
resistivity of the combination is masses in the ratio 3 : 16. The ratio of
n n  1 n 1 n 1 2n resistance of two wires is
1) 2) 3) 4) n  1 1) 3/4 2) 1:2 3) 2:1 4) 4:3
2 2 2n
61. An Aluminium (  = 4 x 10 K ) resistance
-3 -1 68. A wire of resistance 18 ohm is drawn until
R1 and a carbon (  = -0.5 x 10-3K-1) 1
resistance R2 are connected in series to its radius reduce th of its original radius
2
have a resultant resistance of 36  at all then resistance of the wire is
temperatures. The values of R1 and R2 in
 respectively are : 1) 188  2) 72  3) 288  4) 388 
1) 32, 4 2) 16, 20 69. A piece of wire of resistance 4 is bent
3) 4, 32 4) 20, 16 through 1800 at its midpoint and the two
62. The temperature coefficient of a wire is halves are twisted together. Then the re-
0.00125°C–1. At 300 K its resistance is one ohm. sistance is
The resistance of the wire will be 2  at 1) 8  2) 1  3) 2  4) 5 
1) 1154 K 2) 1100 K 70. If three wires of equal resistance are given
3) 1400 K 4) 1127 K then number of combinations they can be
63. The electrical resistance of a mercury made to give different resistance is
column in a cylindrical container is ‘R’. The
mercury is poured into another cylindrical 1) 4 2) 3 3) 5 4) 2
container with half the radius of cross- 71. The effective resistance between A and B
section. The resistance of the mercury in the given circuit is
column is
1) R 2) 2R 3) 16R 4) 5R
64. Four conductors of same resistance
connected to form a square. If the resistance
between diagonally opposite corners is 8
ohm, the resistance between any two
adjacent corners is 1) 20  2) 7  3) 3  4) 6 
1) 32 ohm 2) 8 ohm 72. How many cells each marked 6V  12 A
3) 1/6 ohm 4) 6 ohm
should be connected in mixed grouping so
65. Four wires made of same material have
different lengths and radii, the wire having that it may be marked 24V  24 A
more resistance in the following case is 1) 4 2) 8 3) 12 4) 6
1) A  100cm, r  1mm
73. The effective resistance in series combination
2) A  50cm, r  2mm of two equal resistance is ‘s’. When they
1 are joined in parallel the total resistance
3) A  100 cm , r  mm
is p. If s = np then the minimum possible
2
1 value of ‘n’ is
4) A  50cm, r  mm 1) 4 2) 1 3) 2 4) 3
2

184 | CURRENT ELECTRTICITY |


ACTIVE SITE EDUTECH-9844532971

Sr | 12TH CLASS |NEET PHYSICS : VOL-1|


74. Find out the value of current through 2 78. In the given circuit, the potential of the
resistance for the given circuit point E is

1) 5A 2) 2A 3) zero 4) 4A
75. The resistance of the network between the
terminals A and B is 1) zero 2) –8V 3) –4/3V 4) 4/3V
79. A uniform wire of resistance 20  having
resistance 1  / m is bent in the form of
circle as shown in fig. If the equivalent
resistance between M and N is 1.8  ,
then the length of the shorter section is
1) 30 2) 20
3) 50 4) 60
76. In the figure, the value of resistance to be
connected between C and D so that the
resistance of the entire circuit between A
and B does not change with the number of
elementary sets used is
1) 2 m 2) 5 m 3) 1.8 m 4) 18 m
80. If the voltmeter reads 0.2 V and the
ammeter reads 0.101A, the resistance of
the voltmeter is (in ohm)

1) R 2) R  3 1 3) 3R 4) R  3 1
77. The effective resistance across the points
A and I is

1) 500 2) 1000 3) 200 4) 400


81. In the given circuit Ammeter reading is
same when both switches S1, S2 are closed
or opened. The value of resistance R is

1) 2 2) 1 3) 0.5  4) 5 
1) 200  2) 100  3) 400  4) –300 

| CURRENT ELECTRTICITY | 185


ACTIVE SITE EDUTECH-9844532971

SrSR||1212THTHCLASS
CLASS|NEET
|NEETPHYSICS
PHYSICS: VOL-1|
: VOL-1|
82. In the following diagram ammeter reading 86. What is the equivalent resistance of the
is 4A, voltmeter reading is 20V, the value circuit
of R is

1)  5 2)  5 3)  5 4)  5
1) 6 2) 7 3) 8 4) 9
83. Twelve resistances each of resistance R
are connected in the circuit as shown in fig. 87. The temperature coefficient of
Net resistance between points A and C resistance of platinum is
would be   3.92  10 K at 20 C . Find the
3 1 0

temperature at which the increase in the


resistance of platinum wire is 10% of
its value at 200 C
1) 40.50 C 2) 45.50 C
3) 48.50 C 4) 43.50 C
88. Four identical resistance are joined as
shown in fig. The equivalent resistance
between points A and B is R1 and that
R1
between A and C is R2. Then ratio of R is
2

5R 7R 3R
1) 2) 3) R 4)
3 6 4
84. A resistance is made by connecting two
wires (series) of same material of radii
2 mm and 5 mm and length 8 cm and 5 cm.
A potential difference of 22V is applied to
them. The potential difference on the
1) 1: 5 2) 3:4 3) 2:5 4) 1:2
longer wire is
89. If the galvanometer reading is zero in the
1) 15 V 2) 18 V 3) 16 V 4) 20 V
given circuit, the current passing through
85. A 220 V and 800 W electric kettle and resistance 250  is
three 220V and 100W bulbs are connected
in parallel. On connecting this combination
with 200 V supply, the total current in the
circuit will be
1) 0.15 A 2) 5A 1) 0.016 A 2) 0.16 A
3) 5.5A 4) 4.55A
3) 0.032A 4) 0.042 A
186 | CURRENT ELECTRTICITY |
ACTIVE SITE EDUTECH-9844532971

Sr | 12TH CLASS |NEET PHYSICS : VOL-1|


90. The effective resistance between A and B 95. Three resistances of equal values are
is the given circuit is arranged in four different configurations
as shown below. Power dissipation in the
increasing order is (E-2012)

(I)

1) 3  2) 2  3) 4  4) 6 
91. The equivalent resistance between points
A and B of an infinite network of resistance
each of 1  connected as shown is

1 5 2 5 3 5 1 7
1) 2) 3) 4)
2 4 2 3
92. Equivalent resistance across A and B in
the given circuit is

1) (III) < (II) < (IV) < (I)


2) (II)<(III)<(IV)<(I)
2r 8r 7r 3) (I) < (IV) < (III) < (II)
1) 2) 3) 4) 6r
3 7 3 4) (I)<(III)<(II)<(IV)
93. Two resistances of 400  and 800  are 96. If 400  of resistance is made by adding
connected in series with 6V battery of four 100  resistances of tolerance 5%,
negligible internal resistance. A voltmeter
then the tolerance of the combination is
of resistance 10,000  is used to measure [Mains-2011]
the p.d. across 400  . The error in the 1) 5% 2) 10% 3) 15% 4) 20%
measurement of p.d. in volts approximately 97. In the circuit shown, the current in the 1
1) 0.05 V 2) 0.5 V 3) 0.75 V 4) 5 V resistor is: (Jee 2015 Mains)
94. Copper and carbon wires are connected in
series and the combined resistor is kept
at 0°C. Assuming the combined resistance
does not vary with temperature, the ratio
of the resistances of carbon and copper
wires at 0°C is (Temperature coefficients
of resistivity of copper and carbon
respectively are 4 × 10–3/°C and – 0.5 ×
1) 1.3A, from P to Q 2) 0A
10–3/°C) (M-2013)
3) 0.13A, form Q to P 4) 0.13A, from P to Q
1) 2 2) 4 3) 8 4) 6

| CURRENT ELECTRTICITY | 187


ACTIVE SITE EDUTECH-9844532971

SrSR||1212THTHCLASS
CLASS|NEET
|NEETPHYSICS
PHYSICS: VOL-1|
: VOL-1|
98. The temperature dependence of 101. Two bulbs are fitted in a room in the
restistances of Cu and undoped Si in the domestic electric installation. If one of them
temperature range 300-400K, is best glows brighter than the other, then
described by (Jee Mains 2016) 1) the brighter bulb has smaller resistance
1) Linear increase for Cu, exponential 2) the brighter bulb has larger resistance
decrease for Si 3) both the bulbs have the same resistance
2) Linear decrease for Cu, linear decrease for 4) nothing can be said about the resistance
Si unless other factors are known
3) Linear increase of Cu, linear increase for 102. Three identical bulbs P, Q and R are
Si connected to a battery as shown in the
4) Linear increase for Cu, exponential increase figure. When the circuit is closed
for Si

20) 2 21) 1 22) 1 23) 4 24) 1 25) 3


26) 1 27) 3 28) 1 29) 1 30) 2 31) 4
32) 1 33) 3 34) 3 35) 2 36) 1 37) 1
38)4 39) 1 40) 4 41) 342) 2 43) 1
44) 4 45) 1 46) 3 47) 3 48) 4 49) 3
50) 1 51) 4 52) 4 53) 3 54) 1 55) 2
56) 4 57) 2 58) 3 59) 4 60) 2 61) 3
62) 4 63) 3 64) 4 65) 3 66) 2 67) 4 1) Q and R will be brighter than P
68) 3 69) 2 70) 1 71) 4 72) 1 73) 1 2) Q and R will be dimmer than P
74) 3 75) 2 76) 2 77) 2 78) 3 79) 1 3) All the bulbs will be equally bright
80) 3 81) 4 82) 1 83) 4 84) 4 85) 4 4) Q and R will not shine at all
86)3 87) 2 88) 2 89) 1 90) 2 91) 1
92) 2 93) 1 94) 3 95) 1 96) 1 97) 3 103. Figure shows three similar lamps L1, L2, L3
98) 1 connected across a power supply. If the
lamp L3 fuses. The light emitted by L1 and
L2 will change as
ELECTRIC POWER
99. A heater coil is cut into two equal parts and
only one part is used in the heater. Then
the heat generated becomes
1) become one fourth 2) halved
3) doubled 4) become four times
100. Two lamps have resistance r and R, R
being greater than r. If they are connected
in parallel in an electric circuit, then
1) the lamp with resistance R will shine more
brightly 1) no change
2) the lamp with resistance r will shine more 2) brilliance of L1 decreases and that of L2
brightly increases
3) the two lamps will shine equal brightly 3) brilliance of both L1 and L2 increases
4) the lamp with resistance R will not shine at 4) brilliance of both L1 and L2 decreases
all

188 | CURRENT ELECTRTICITY |


ACTIVE SITE EDUTECH-9844532971

Sr | 12TH CLASS |NEET PHYSICS : VOL-1|


104. The potential difference across a conductor 110. A constant voltage is applied between the
is doubled, the rate of generation of heat two ends of a metallic wire. If both the
will length and the radius of the wire are
1) become one fourth 2) be halved doubled, the rate of heat developed in the
3) be doubled times 4) become four times wire
105. Two metallic wires of same material and 1) will be doubled 2) will be halved
same length have different diameters. 3) will remain the same 4) will be quadrupled
When the wires are connected in parallel 111. A resistor R1 dissipates the power P when
across an ideal battery the rate of heat
connected to a certain generator. If the
produced in thinner wire is Q1 and that in
thicker wire is Q2. The correct statement is resistor R2 is put in series with R1, the
power dissipated by R1
1) Q1 = Q2 2) Q1 < Q2 3) Q1 > Q2
1) Decreases 2) Increases
4) It will depend on the emf of the battery
3) Remains the same
106. There are two metallic wires of same
material, same length but of different radii. 4) Any of the above depending upon the relative
When these are connected to an ideal values of R1 and R2
battery in series, heat produced is H1 but
when connected in parallel, heat produced
is H2 for the same time. Then the correct
statement is 99) 3 100)2 101)1 102) 2 103) 2
1) H1  H2 2) H1  H2 104) 4 105) 2 106) 2 107) 3 108) 4
3) H1  H2 4) No relation 109) 4 110) 1 111)1
107. Two electric bulbs rated
ELECTRIC POWER & JOULES LAW
P1 watt and V volt , are connected in
series, across V-volt supply. The total 112. A 25 watt, 220 volt bulb and a 100 watt,
power consumed is 220 volt bulb are connected in series across
P1  P2
440 volt line
1) 2) P1  P2 1) only 100 watt bulb will fuse
2
P1  P2
2) only 25 watt bulb will fuse
3) P  P 4) P1  P2  3) none of the bulb will fuse
1 2
108. In above question, if the bulbs are 4) both bulbs will fuse
connected in parallel, total power 113. There are 5 tube-lights each of 40W in a
consumed is house. These are used on an average for 5
P1  P2 hours per day. In addition, there is an
1) 2) P1  P2 immersion heater of 1500W used on an
2
average for 1 hour per day. The number of
P1  P2
3) P  P 4) P1  P2  units of electricity are consumed in a month is
1 2
1) 25 units 2) 50 units
109. Which of the following causes production
3) 75 units 4) 100 units
of heat, when current is set up in a wire
114. Three equal resistors connected in series
1) Fall of electron from higher orbits to lower
across a source emf together dissipate 10
orbits
watt. If the same resistors are connected
2) Inter atomic collisions in parallel across the same emf the power
3) Inter electron collisions dissipate will be
4) Collisions of conduction electrons with
1) 10 watt 2) 30 watt
atoms
3) 10/3 watt 4) 90 watt

| CURRENT ELECTRTICITY | 189


ACTIVE SITE EDUTECH-9844532971

SrSR||1212THTHCLASS
CLASS|NEET
|NEETPHYSICS
PHYSICS: VOL-1|
: VOL-1|

115. Time taken by a 836 W heater to heat one 124. A 50 C rise in the temperature is
litre of water from 100 C to 400 C is observed in a conductor by passing some
1) 50 s 2) 100 s 3) 150 s 4) 200 s current. When the current is doubled,
then rise in temperature will be equal to
116. A lamp of 600W-240V is connected to
220V mains. Its resistance is 1) 50 C 2) 100 C 3) 200 C 4) 40 0 C
1) 96  2) 84  3) 90  4) 64  125. Two wires A and B with lengths in the
117. A 200W - 200V lamp is connected to 250V ratio of 3 : 1, diameters in the ratio of 1:2
mains. It power consumption is and resistivities in the ratio of 1:20 are
1) 300 W 2) 312.5W 3) 292 W 4) 250 W joined in parallel with a source of emf, 2V.
118. If the current in a heater increases by Ratio of the R1 / R2 is:
10%, the percentage change in the power
consumption 1) 5 : 2 2) 2 : 5 3) 5 : 3 4) 3 : 5
1) 19% 2) 21% 3) 25% 4) 17% 126. An electric heater operating at 220 volts
119. The power of a heating coil is P. It is cut boils 5 litre of water in 5 minutes. If it is
into two equal parts. The power of one of used on 110 volts, it will boil the same
them across same mains is amount of water in
1) 2P 2) 3P 3) P/2 4) 4P
1) 10 minutes 2) 20 minutes
120. In a house there are four bulbs each of
50W and 5 fans each of 60W. If they are 3) 15 minutes 4) 25 minutes
used at the rate of 6 hours a day, the
127. Three electric bulbs of 40W, 60W and
electrical energy consumed in a month of
100W have the tungsten wire of the same
30 days is
diameter. Then the longer wire is used by
1) 64 KWH 2)90.8KWH
3)72 KWH 4) 42 KWH 1) 60W 2) 100W 3) 40W
121. An electric kettle has two coils. When one 4) All use the same length
coil is switched on it takes 15 minutes and
128. A fuse wire with radius of 0.2 mm
the other takes 30 minutes to boil certain
blows off with a current of 5 Amp. The
mass of water. The ratio of times taken by
them, when connected in series and in fuse wire of same material but of radius
parallel to boil the same mass of water is 0.3 mm will blow off with a current of
1) 9 :2 2) 2:9 3) 4:5 4) 5:4
3 5 3
122. A resistance coil of 60  is immersed in 1) 5  amp 2) amp
2 2
42kg of water. A current of 7A is passed
through it. The rise in temperature of water 27
per minute is 3) 5 amp 4) 5 amp
8
1) 40 C 2) 80 C 3) 1.30C 4) 120 C
123. What is the required resistance of the 129. In a large building, there are 15 bulbs of 40
heater coil of an immersion heater that W, 5 bulbs of 100 W, 5 fans of 80 W and 1
will increase the temperature of 1.50 kg heater of 1 kW. The voltage of electric
of water from 100 C to 500 C in 10 mains is 220 V. The minimum capacity of
the main fuse of the building will be:
minutes while operating at 240V ?
[Mains-2014]
1) 25 2) 12.5
1) 8 A 2) 10 A 3) 12 A 4) 14 A
3) 250 4) 137.2

190 | CURRENT ELECTRTICITY |


ACTIVE SITE EDUTECH-9844532971

Sr | 12TH CLASS |NEET PHYSICS : VOL-1|


130. The supply voltage to room is 120 V. The 136. From the following the standard cell is
resistance of the lead wires is 6  . A 60 W 1) Daniel cell 2) Cadmium cell
bulb is already switched on. What is the
3) Leclanche cell 4) Lead accumulator
decrease of voltage across the bulb, when
a 240 W heater is switched on in parallel 137. A cell is to convert
to the bulb? [Mains-2013] 1) chemical energy into electrical energy
1) zero 2) 2.9 Volt 3) 13.3 Volt 4) 10.04 Volt 2) electrical energy into chemical energy
3) heat energy into potential energy
4) potential energy into heat energy
112) 2 113) 3 114) 4 115)3 116) 1 138. ‘n’ identical cells, each of internal
117) 2 118) 2 119) 1 120) 2 121) 1 resistance (r) are first connected in parallel
122) 3 123) 4 124) 3 125) 4 126) 2 and then connected in series across a
127) 3 128) 3 129) 3 130) 4 resistance ( R). If the current through R
is the same in both cases, then
1) R = r/2 2) r = R/2 3) R = r 4) r = 0
CELLS-INTERNAL RESISTANCE & EMF
139. The value of internal resistance of ideal cell is
131. Back emf of a cell is due to 1) Zero 2) infinite 3) 1  4) 2 
1) Electrolytic polarization 140. In a circuit two or more cells of the same
2) Peltier effect emf are connected in parallel in order
3) Magnetic effect of current 1) Increases the pd across a resistance in the
4) Internal resistance circuit
132. The direction of current in a cell is
2) Decreases pd across a resistance in the
1)   ve pole to   ve pole during circuit
discharging 3) Facilitate drawing more current from the
battery system
2)   ve pole to   ve pole during
4) Change the emf across the system of
discharging batteries
3) Always   ve pole to   ve pole 141. The resistance of an open circuit is
4) always flows from (+)ve ploe to (-) ve pole 1) Infinity 2) Zero
133. When an electric cell drives current
3) Negative 4) cann’t be predicted
through load resistance, its Back emf,
1) Supports the original emf 142. According to joule's law if potential
2) Opposes the original emf difference across a conductor having a
3) Supports if internal resistance is low material of specific resistance  , remains
4) Opposes if load resistance is large constant, then heat produced in the
134. The terminal voltage of a cell is greater conductor is directly proportional to
than its emf. when it is 1 1
1) being charged 2) an open circuit 1)  2)  2 3) 4)
3) being discharged 4) it never happens  
135. What is constant in a battery ( also called 143. Internal resistance of a cell depends on
a source of emf) ? 1) concentration of electrolyte
1) current supplied by it
2) distance between the electrodes
2) terminal potential difference
3) internal resistance 3) area of electrode
4) emf 4) all the above

| CURRENT ELECTRTICITY | 191


ACTIVE SITE EDUTECH-9844532971

SrSR||1212THTHCLASS
CLASS|NEET
|NEETPHYSICS
PHYSICS: VOL-1|
: VOL-1|
144. When cells are arranged in series
1) the current capacity decreases 10 20
1) V 2) V 3) 10V 4) 5V
2) The current capacity increases 3 3
3) the emf increases 151. In the following diagram, the pd across 6V
4) the emf decreases cell is
145. To supply maximum current, cells should
be arrange in
1) series 2) parallel 3) Mixed grouping
4) depends on the internal and external
resistance
146. The terminal Pd of a cell is equal to its emf if
1) external resistance is infinity
2) internal resistance is zero
1) 6V 2) 5.6V 3) 8.2V 4) 8.4V
3) both 1 and 2 4) internal resistance is 5
147. The electric power transffered by a cell to 152. While connecting 6 cells in a battery in
an external resistance is maximum when series, in a tape recorder, by mistake one
the external resistance is equal to ...(r cell is connected with reverse polarity. If
internal resistance) the effective resistance of load is 24 ohm
and internal resistance of each cell is one
r ohm and emf 1.5V, the current delivered
1) 2) 2r 3) r 4) r2
2 by the battery is
148. Which depolarizers are used to neutralizes 1) 0.1A 2) 0.2A 3) 0.3A 4) 0.4A
hydrogen layer in cells 153. A 10m long wire of resistance 15 ohm is
1) Potassium dichromate connected in series with a battery of emf 2V
2) Manganese dioxide (no internal resistance) and a resistance of
3) 1 or 2 4) hydrogen peroxide 5 ohm. The potential gradient along the wire
149. The potential difference between A and B is is
1) 0.15 Vm-1 2) 0.45V m-1
3) 1.5Vm-1 4)4.5Vm-1
154. When a resistance of 2 ohm is placed
across a battery the current is 1A and when
the resistance across the terminals is 17
ohm, the current is 0.25A. the emf of the
battery is
1) 4.5 V 2) 5 V 3) 3 V 4) 6 V
1) 9V 2) 15V 3) 10V 4) 20V 155. A battery has six cells in series. Each has
150. The p.d. across the resistance of 400 ohm, an emf 1.5V and internal resistance 1 ohm.
as will be measured by the voltmeter V of If an external load of 24  is connected
resistance 400 ohm to it. The potential drop across the load is
1) 7.2V 2) 0.3V 3) 6.8V 4) 0.4V
156. 12 cells of each emf 2V are connected in
series among them, if 3 cells are connected
wrongly. Then the effective emf. of the
combination is
1) 18 V 2) 12 V 3) 24 V 4) 6 V

192 | CURRENT ELECTRTICITY |


ACTIVE SITE EDUTECH-9844532971

Sr | 12TH CLASS |NEET PHYSICS : VOL-1|

157. When a battery connected across a 161. Twenty four cells each of emf 1.5V and
resistor of 16  , the voltage across the internal resistance 0.5 ohms are to be
resistor is 12V.When the same battery is connected to a 3 ohm resistance. For
maximum current through this resistance
connected across a resistor of 10  ,
the number of rows and number of columns
voltage across it is 11V. The internal that you connect these cells is.
resistance of the battery in ohms is
1) 12 cells in series 2 rows in parallel
1) 10/7 2) 20/7 3) 25/7 4) 30/7 2) 8 cells in series 3 rows in parallel
158. In the circuit shown here, cells A and B 3) 4 cells in series 6 rows in parallel
have emf 10 V each and the internal
4) 6 cells in series 4 rows in parallel
resistance is 5 for A and 3 for B. For
162. A battery of four cells in series each having
what value of R will the potential difference
an emf of 1.5V and internal resistance 1 
across the cell A will be zero?
are connected in series with an ammeter,
a coil of resistance 2  and a filament
lamp. If the ammeter reads 0.5A, the
resistance of the filament lamp is
1) 4  2) 6  3) 2  4) 12 
163. A 5V battery with internal resistance
1) zero 2) 1 ohm 2  and a 2V battery with internal resistance
3) 2 ohm 4) 3 ohm 1  are connected in parallel with unlike
159 In the circuit of fig. with steady current, polarities connected together .This
the potential drop across the capacitor is combination is connected to 10  resistor
The current in the 10  resistor is
1) 0.27 A 2) 0.05 A 3) 0.25 A 4) 0.3 A
164.A voltmeter resistance 500  is used
to measure the emf of a cell of internal
resistance 4  . The percentage error
in the reading of the voltmeter will be
V V 2V
1) V 2) 3) 4) 1) 0.4% 2) 0.6% 3) 0.8% 4) 1.2%
2 3 3
165. When two identical cells are connected
160. In the circuit, the galvanometer G shows either in series or in parallel across a 4 ohm
zero deflection. If the batteries A and B resistor, they send the same current
have negligible internal resistance, the through it. The internal resistance of the
value of the resistor R will be : cell in ohm is
1) 4  2) 2  3) 1  4) 7 
166. Two cells with same e.m.f. ‘E’ and differ-
ent internal resistances r1 and r2 are con-
nected in series to an external resistance
‘R’. The value of R so that the p.d. across
1) 100 2) 200
the first cell be zero is
3) 500 4) 1000
r1  r2
1) r2  r1 2) r1  r2 3) r1  r2 4)
2
| CURRENT ELECTRTICITY | 193
ACTIVE SITE EDUTECH-9844532971

SrSR||1212THTHCLASS
CLASS|NEET
|NEETPHYSICS
PHYSICS: VOL-1|
: VOL-1|
167. Two conductors have the same resistance 172. In a balanced Wheatstone’s network, the
at 0°C but their temperature coefficients resistances in the arms Q and S are
interchanged. As a result of this :
of resistance are 1 and  2 . The
1)galvanometer and the cell must be
respective temperature coefficients of interchanged to balance
their series and parallel combinations are
2) galvanometer shows zero deflection
nearly [AIE-2010]
3) network is not balanced
1   2 1   2 4) network is still balanced
1) , 1   2 2) 1   2 ,
2 2 173. If galvanometer and battery are
1 2 interchanged in balanced Wheatstone’s
   , 1   2 1   2
3) 1 2 4) , bridge, then
1   2 2 2
1) the battery discharges
168. A galvanometer having a coil of resistance
2) the bridge still balances
100 gives a full scale deflection, when a
current of 1mA is passed through it. The 3) the balance point is changed
value of the resistance, which can convert 4) the galvanometer is damaged due to flow of
this galvanometer into ammeter giving a high current
full scale deflection for a current of 10A, 174. Wheatstone bridge can be used
is (Jee Mains 2016)
1) To compare two unknown resistances.
1) 0.1 2) 3 3) 0.01 4) 2
2) to measure small strains produced in hard
metals
3) as the working principle of meter bridge
4) All the above
131) 1 132) 1 133) 2 134) 1 135) 4
136) 2 137) 1 138) 3 139) 1 140) 3 175. In a Wheatstone's bridge three
141) 1 142)4 143) 4 144) 3 145) 4 resistances P,Q,R connected in three arms
146) 3 147) 3 148) 3 149) 1 150) 2 and the fourth arm is formed by two
151) 4 152) 2 153) 1 154) 2 155) 1 resistances S1,S2 connected in parallel.
156) 2 157) 2 158)3 159) 3 160) 2 The condition for bridge to be balanced will
161) 1 162) 2 163) 3 164) 3 165) 1 be
166) 2 167) 4 168) 3
P R P 2R
1) Q  S  S 2) Q  S  S
KIRCHOFF’S LAWS 1 2 1 2

WHEATSTONE BRIDGE P R ( S1  S 2 ) P R ( S1  S 2 )
3) Q  S S 4) Q  2 S S
169. Kirchhoff’s law of meshes is in accordance 1 2 1 2

with law of conservation of 176. Assertion : At any junction of a network,


1) charge 2) current algebraic sum of various currents is zero
3) energy 4) angular momentum Reason : At steady state there is
170. Kirchhoff’s law of junctions is also called no accumulation of charge at the junction.
the law of conservation of 1) Both (A) and (R) are true and (R) is the
correct explanation of A.
1) energy 2) charge
2) Both (A) and (R) are true but (R) is not the
3) momentum 4) angular momentum correct explanation of A.
171. Wheatstone’s bridge cannot be used for 3) (A) is true but (R) is false
measurement of very ——— resistances. 4) (A) is false but (R) is true
1) high 2) low 3) low(or) high 4) zero

194 | CURRENT ELECTRTICITY |


ACTIVE SITE EDUTECH-9844532971

Sr | 12TH CLASS |NEET PHYSICS : VOL-1|


177. Six resistors of each 2 ohm are connected 288 8 9
as shown in the figure. The resultant 1)  2) 12  3)  4) 
resistance between A and B is. 56 3 4
182. In the circuit shown in the figure, the value
of Resistance X, when potential difference
between the points B and D is zero will be
1) 9  2) 8  3) 6  4) 4 
183. The electric current i in the circuit shown
is (E-2011)
1) 4  2) 2  3) 1  4) 10 
178. In the given circuit current through the
galvanometer is

1) 6 A 2) 2 A 3) 3 A 4) 4 A
184. In the circuit shown in the figure, the
current ‘I’ is
(EAM-2013)
1) Zero 2) Flows from C to D
3)Flows from D to C 4)In sufficient information
179. The potential difference between A & B in
the given branch of a circuit is

1) 6 A 2) 2 A 3) 4 A 4) 7 A
1) 6V 2) 12V 3) 9V 4) 0V
185. Four resistors A, B, C and D form a
180. The resistance between A and B is Wheatstone’s bridge. The bridge is
balanced when C = 100  . If A and B are
inter changed, the bridge balances for C =
121  . The value of D is (E-2012)
1) 10  2) 100  3) 110  4) 200 
1) 8  2) 4  3) 3.75  4) 2  186. In the circuit shown below, the ammeter
reading is zero. Then the value of the
181. The resistance between A and B is resistance R is (E-2011)

1) 50  2) 100 
3) 200  4) 400 

| CURRENT ELECTRTICITY | 195


ACTIVE SITE EDUTECH-9844532971

SrSR||1212THTHCLASS
CLASS|NEET
|NEETPHYSICS
PHYSICS: VOL-1|
: VOL-1|

10 12 2 12
1) A, A 2)  A, A
7 7 7 5
169) 3 170) 2 171) 2 172) 3 173) 2
174) 4 175) 3 176) 1 177) 3 178) 3 8 12 8 12
3) A, A 4) A, A
179) 1 180) 2 181) 3 182) 2 183) 4 7 5 7 7
184) 3 185) 3 186) 2 192. When an unknown resistance and a
METERBRIDGE resistance of 4  are connected in the left
187. Metal wire is connected in the left gap, semi and right gaps of a Meter bridge, the
conductor is connected in the right gap of balance point is obtained at 50cm. The shift
meter bridge and balancing point is found. in the balance point if a 4  resistance is
Both are heated so that change of now connected in parallel to the resistance
resistances in them are same. Then the in the right gap is
balancing point
1) 66.7cm 2)16.7 cm
1) will not shift 2) shifts towards left
3) 34.6 cm 4) 14.6 cm
3) shifts towards right
193. In a meter bridge, the gaps are closed by
4) depends on rise of temperature
resistances 2 and 3 ohms. The value of
188. A meter bridge is balanced with known shunt to be added to 3 ohm resistor to shift
resistance in the right gap and a metal wire
the balancing point by 22.5 cm is
in the left gap. If the metal wire is heated
the balance point. 1) 1  2) 2  3) 2.5  4) 5 
1) shifts towards left 2) shifts towards right 194. Two equal resistance are connected in the
3) does not change gaps of a meter bridge. If the resistance in
4) may shift towards left or right depending on the left gap is increased by 10%, the
the nature of the metal. balancing point shift
189. In meter bridge experiment of resistances, 1) 10 % to right 2) 10% to left
the known and unknown resistances are 3) 9.6% to right 4) 4.8% to right
inter-changed . The error so removed is
195. Two unknown resistance X and Y are
1) end correction 2) index error connected to left and right gaps of a meter
3) due to temperature effect 4) random error bridge and the balancing point is obtained
190. In a meter bridge experiment, when the at 80 cm from left. When a 10  resistance
resistances in the gaps are interchanged,
is connected in parallel to ‘x’, the balance
the balance-point did not shift at all. The
ratio of resistances must be point is 50 cm from left. The values of X
and Y respectively are
1) Very large 2) Very small
3) Equal to unity 4) zero 1) 40 , 9 2) 30 , 7.5

191. In the given circuit currents I1 and I 2 3) 20 , 6 4) 10 , 3


196. In the meter bridge experiment, the length
AB of the wire is 1 m. The resistors X and
Y have values 5  and 2  respectively.
When a shunt resistance S is connected to
X, the balancing point is found to be 0.625
m from A. Then, the resistance of the shunt
is (Eam-2013)

196 | CURRENT ELECTRTICITY |


ACTIVE SITE EDUTECH-9844532971

Sr | 12TH CLASS |NEET PHYSICS : VOL-1|


201. If the emf of the cell in the primary circuit
is doubled, with out changing the cell in the
secondary circuit, the balancing length is
1) Doubled 2) Halved
3) Uncharged 4) Zero
202. The potential gradients on the
potentiometer wire are V1 and V2 with an
1) 5  2) 10  3) 7.5  4) 12.5  ideal cell and a real cell of same emf in the
primary circuit then
1) V1  V2 2) V1  V2
187) 3 188) 2 189) 1 190) 3 191) 2
3) V1  V2 4) V1  V2
192) 2 193) 2 194) 4 195) 2 196) 2
203. If the current in the primary circuit is
POTENTIOMETER decreased, then balancing length is
obtained at
197. A potentiometer is superior to voltmeter 1) Lower length 2) Higher length
for measuring a potential because
1) voltmeter has high resistance 3) Same length 4) 1/3rd length
2) resistance of potentiometer wire is quite 204. Temperature coefficient of resistance ’ ’
low and resistivity ‘  ’ of a potentiometer wire
3) potentiometer does not draw any current must be
from the unknown source of emf. to be 1) high and low 2) low and high
measured.
4) sensitivity of potentiometer is higher than 3) low and low 4) high and high
that of a voltmeter. 205. A series high resistance is preferable than
198. In comparing emf’s of 2 cells with the help shunt resistance in the galvanometer
of potentiometer, at the balance point, the circuit of potentiometer. Because
current flowing through the wire is taken from 1) shunt resistances are costly
1) Any one of these cells. 2) shunt resistance damages the galvanometer
2) both of these cells
3) series resistance reduces the current through
3) Battery in the primary circuit
galvanometer in an unbalanced circuit
4) From an unknown source
199. A potentiometer wire is connected across 4) high resistances are easily available
the ideal battery now, the radius of 206. The sensitivity of potentiometer wire can
potentiometer wire is doubled without be increased by
changing its length. The value of potential 1) decreasing the length of potentiometer wire
gradient
1) increases 4 times 2) increases two times 2) increasing potential gradient on its wire
3) Does not change 4) becomes half 3) increasing emf of battery in the primary circuit
200. In a potentiometer of ten wires, the 4) decreasing the potential gradient on its wire
balance point is obtained on the sixth wire. 207. A cell of emf ‘E’ and internal resistance ‘r’
To shift the balance point to eighth wire, connected in the secondary gets balanced
we should against length ‘ A ’ of potentiometer wire.
1) increase resistance in the primary circuit. If a resistance ‘R’ is connected in parallel
2) decrease resistance in the primary circuit. with the cell, then the new balancing length
3) decrease resistance in series with the cell for the cell will be
whose emf. has to be measured.  R ­¬ Rr R  R 
4) increase resistance in series with the cell 1) žžžŸ ­ l 2)   l 3)   4)  l
R  r ­®  R  r Rr
whose emf. has to be measured.

| CURRENT ELECTRTICITY | 197


ACTIVE SITE EDUTECH-9844532971

SrSR||1212THTHCLASS
CLASS|NEET
|NEETPHYSICS
PHYSICS: VOL-1|
: VOL-1|
208. Potentiometer is an ideal instrument, 215. The quantity that cannot be measured by
because a potentiometer is ...........
1) no current is drawn from the source of 1) Resistance 2) emf
unknown emf 3) current in the wire 4) Inductance
2) current is drawn from the source of unknown 216. A potentiometer having a wire of 4m length
emf
is connected to the terminals of a battery
3) it gives deflection even at null point
with a steady voltage. A Leclanche cell
4) it has variable potential gradient
has a null point at 1m. If the length of the
209. On increasing the resistance of the primary
circuit of potentiometer, its potential potentiometer wire is increased by 1m, The
gradient will position of the null point is
1) become more 2) become less 1) 1.5m 2) 1.25m
3) not change 4) become infinite 3) 10.05m 4) 1.31m
210. If the value of potential gradient on 217. The emf of a battery A is balanced by a
potentiometer wire is decreased, then the length of 80cm on a potentiometer wire.
new null point will be obtained at The emf of a standard cell 1v is balanced
1) lower length 2) higher length by 50cm. The emf of A is
3) same length 4) nothing can be said 1) 2 V 2) 1.4 V 3) 1.5 V 4) 1.6 V
211. A cell of negligible internal resistance is 218. When 6 identical cells of no internal resistance
connected to a potentiometer wire and are connected in series in the secondary
potential gradient is found. Keeping circuit of a potentiometer, the balancing
the length as constant, if the radius of length is ‘ l ’, balancing length becomes
potentiometer wire is increased four
times, the potential gradient will become l /3 when some cells are connected wrongly,
(no series resistance in primary) the number of cells connected wrongly are
1) 4 times 2) 2 times 3) half 4) constant 1) 1 2) 3 3) 2 4) 4
212. For the working of potentiometer, the emf of 219. In a potentiometer experiment, the balancing
cell in the primary circuit (E) compared to the length with a cell is 560cm. When an
emf of the cell in the secondary circuit (E1) is external resistance of 10ohms is connected
1) E > E1 2) E < E1 in parallel to the cell the balancing length
3) Both the above 4) E = E1 changes by 60cm. The internal resistance
213. The balancing lengths of potentiometer of the cell in ohm is
wire are l1 and l2 when two cells of emf E1 1) 3.6 2) 2.4 3) 1.2 4) 0.6
and E2 are connected in the secondary
220. The resistivity of a potentiometer wire is,
circuit in series first to help each other and
if the area of cross section of the wire is
E1
next to oppose each other E is equal to 4cm2. The current flowing in the circuit is
2 1A, the potential gradient is 7.5 v/m
(E1>E2).
1) 3 × 10–3  - m 2) 2 × 10–6  - m
l1 l1  l2 l1  l2 l2 3) 4 × 10–6  - m 4) 5 × 10–4  - m
1) l 2) l  l 3) l  l 4) l
2 1 2 1 2 1 221. A potentiometer wire of 10m length and 20
214. At the moment when the potentiometer is Ohm resistance is connected in series with
balanced, a resistance R ohms and a battery of emf
1) Current flows only in the primary circuit 2V, negligible internal resistance,
2) Current flows only in the secondary circuit Potential gradient on the wire is 0.16 milli-
3) Current flows both in primary and volt / centimeter then R is ...ohms
secondary circuits
4) Current does not flow in any circuit 1) 50  2) 60  3) 230  4) 46 

198 | CURRENT ELECTRTICITY |


ACTIVE SITE EDUTECH-9844532971

Sr | 12TH CLASS |NEET PHYSICS : VOL-1|


222. The potential gradient along the length of
a uniform wire is 10 volt/m B and C are
two points at 30 cm and 60 cm in a scale
fitted along the wire. The pd between B 197) 3 198) 3 199) 3 200) 1 201) 2
and C is 202) 2 203) 2 204) 2 205) 3 206) 4
1) 3V 2) 0.4V 3) 7V 4) 4V 207) 4 208) 1 209) 2 210) 2 211) 4
223. In the determination of the internal resis- 212) 1 213) 3 214) 1 215) 4 216) 2
tance of a cell using a potentiometer, when 217) 4 218) 3 219) 3 220) 1 221) 3
the cell is shunted by a resistance “R” and 222) 1 223) 2 224) 1 225) 2 226) 3
connected in the secondary circuit, the
balance length is found to be L 1. On
doubling the shunt resistance, the balance
length is found to increase to L2. The value HINTS SOLUTIONS
of the internal resistance is
2 R ( L2  L1 )
1) ( L  2 L )
2 R ( L2  L1 )
2) (2 L  L )
ELECTRIC CURRENT AND DRIFT
1 2 1 2
VELOCITY
R( L2  L1 ) R( L2  L1 )
3) ( L  2 L ) 4) (2 L  L )
1 2 1 2

224.A potentiometer circuit for comparison 10. i  neAvd ;


of two resistances. The balance point
with a standard resistor R  10.0  is 5  5  1026  1.3  1019  4  106 vd
found to be 58.3 cm, while that with the 1
unknown resistance X is 68.5 cm. The vd  m/sec
64
value of X is
5
1) 11.75 2) 12.55 11. i = qf 12. q   idt
0
3) 9.55 4) 12.75
225.In an experiment for calibration of 13. E 
i
14. i 
n e  n p e
A t
voltmeter a standard cell of emf 1.5 V is
balanced at 300 cm length of q ev
t2

potentiometer wire. The P.D across a 15. i   16. q   I . dt


t 2 r
resistance in the circuit is balancedat t1

1.25 m. If a voltmeter is connected 1 i


across the same resistance it reads 17. vd  18. vd 
A neA
0.65 V. The error in the volt meter is
1) 0.5V 2) 0.025V l
19. R  ;
3) 0.05 V 4) 0.25V A
226. The current in the primary circuit of a V l V l V
potentiometer is 0.2 A. The specific    
i A nAeVd A neVd l
resistance and cross-section of the
potentiometer wire are 4 × 10 –7 m 2, 6
respectively. The potential gradient will be 45. Req   3
2
equal to [Mains-2011]
1)1 V/m 2)0.5 V/m 3)0.1 V/m 4)0.2 V/m

| CURRENT ELECTRTICITY | 199


ACTIVE SITE EDUTECH-9844532971

SrSR||1212THTHCLASS
CLASS|NEET
|NEETPHYSICS
PHYSICS: VOL-1|
: VOL-1|
57. combination of resistors
46.
58. R  52  103  10% 59. R  l 2
60. R  R1  R 2  ...  Rn
QO  = 1 l +2
l
+ .....+ n
l
A A A A
n  1  2  3  .........  n
n n  1
n 1 S
n  
2 2
61. R1  1 = R2  2 and R1+R2 = 36 ohm.
R2  R1
62.  
R1t2  R2t1

l
63. R  , Volume V  A.l
A
64. Combination of resistors
l l
65. R . Check the options 66. R 
16 R r 2
A
Req  10 
16  R l2 1
67. R  68. R 
16 R 16 R m r4
18  10  8
16  R 16  R R1 R2
69. R  R  R
1
70. 2n-1
125  8 R  16 R  8 R  128 1 2

71. Combination of resistors


47. 72. Number of rows
requiredcurrent 24A
  2m
Givencurrent 12A

26 24
Parallel,  1.5 requried potential  4n
26  6
Given potential
3 6
Req   ; i   4A  Total no of cells  n  m
2 3/ 2
 2 4  8
48. no of combinations x = 2n
73. RS = n RP(where n is number of resistors)
2
49. combination of resistors
50 combination of resistors
51. combination of resistors
R 74.
R
52. eff 53: R  l 2
n2
54. Solving for effective resistance by series and
parallel combination
56. combination of resistors 75. End resistors are not considered

200 | CURRENT ELECTRTICITY |


ACTIVE SITE EDUTECH-9844532971

Sr | 12TH CLASS |NEET PHYSICS : VOL-1|


77. The equivalent circuit is shown
V
89. i  93. error = V1  V11
Rtotal
94. R11  R2 2 95. P  i 2 R
78.
96. 4 R  400  20
97. Using Kirchhoff’s voltage law.
98. Based on temperature dependence of resistance.
V2
112. R  ; V  iR
P
8 4
VC  0 ; i  A E
5 1 3 113. P  , 1 K.W.H = 1 unit
t
4
VC  VE  i  1  V2 P R
3 114. P   S  P
R PP RS
4 4
0  VE   VE   V 115. W  JQ  P  t  JmsT
3 3
79. Let the resistance of shorter part MN be x. (V ) 2 V2
116. R  117. P 
Then resistance of longer part is (20 – x)  P R
 2x  V2 l
80.  2  x  (0.101) = 0.2 solve for ‘x’ 118. Pi 2 119. P  ; R
  R A
1 .5 no. watts  no. of hours
81. When S1 and S2 are opened, i = ..... (1) 120. Number of units = 1000
450
When S1 and S2 are closed, t1t2
121. ts = t1 + t2 ; tp = t  t
1.5[100  R ] 1 2
i= ....... (2)
400R  30,000 122. JQ  i 2 Rt , mSt  i Rt 2

82. iR  20, here i  4A  R  5 . 123. use Joule’s law


83. When a cell is connected across A and C, no Q  msT  but Q  i 2 RT  T  i 2
current flows in the arms BG and ED due to t i 2
l
symmetry in the arrangement. Then equivalent 124. t  i  t  20 C 125. R  2
1 1 o
2
r
2

circuit will be as shown in Fig.(a) and (b).


2 2

The effective resistance between A and C is V2 t2 V12


84. Apply Ohm’s law. 126. H  t ;  
R t1 V 22
V2 V
85. R  ; find Req then i  R V 2 V 2A
 P
1
P 127. P  
eq
R l l
86. Combination of resistors
128. i 2 r 3
R2  R1
87. t  129. p  v  i
R1
88. Combination of resistors V2
130. P 
R

| CURRENT ELECTRTICITY | 201


ACTIVE SITE EDUTECH-9844532971

SrSR||1212THTHCLASS
CLASS|NEET
|NEETPHYSICS
PHYSICS: VOL-1|
: VOL-1|

30 6 4 2V  V V
  159. I   ;Apply loop law in upper
149. E  6 5 2  5  2  2  9V 3R 3R
1 1 1 1 loop
 
6 3 2
V
IR  V  V  VC  0 or VC  IR  R
3R
E  9V V
or VC 
V A  VB  E  9V 3
160. i = E / r + R

mnE nE
161. i  & m  n  24 162. i 
mR  nr R  nr
150.
2E 2E
163. I s  , and I P 
2r  R r  2R
r=R

E ir
164. % error =  100   100
E E

165. when is  i p ; r = R

Balanced Wheatstone bridge E1  E2


166. i  R  r  r ; V1  E1  i r1
1 2
200 20
V1   10  V
100  200 3 167. R1  R0 (1  1 t )  ;
n  2 m  E R2  R0 (1   2 (t ))
151. V = E + ir 152. i 
R  nr 
1 1 1
E R Series R  R1  R2 ; Parallel R  R  R
153. Potential gradient = r  R  R  L 1 2
S P

2 15 G
=   0.15 168. S  ; 177. Use K.V.L
0  15  5 10 I
1
E  nE  ig
154. i  155. V   R
Rr  R  nr  P R
182. 
156. Eeq   N  2m E Q S
 E  V1   E  V2  183. Applying Kirchhoff’s first law
157. r   V  R1   V  R2 .
 1   2  184. Using Kirchhoff’s law
e1  e2 185. Using Wheatstone’s bridge principle
158. i  R  r  r , v  e1  ir1  0
1 2 186. Applying Kirchhoff’s law

202 | CURRENT ELECTRTICITY |


ACTIVE SITE EDUTECH-9844532971

Sr | 12TH CLASS |NEET PHYSICS : VOL-1|

191.
STUDENT EXERCISE
PRACTICE QUESTIONS
ELECTRIC CURRENT & DRIFT VELOCITY
1. A current of 1.6 A is flowing in a conductor.
B to B : (path BAGFB) The number of electrons flowing per second
2 through the conductor is
VB  I 1  7  6  4  VB ; I 1   A 1) 109 2) 1019 3) 1016 4) 1031
7
B to B : (path BCDFB) 2. If an electron revolves in the circular path of
12 radius 0.5A0 at a frequency of 5 x 1015 cycles/
VB  I 2  5  8  4  VB ; I 2  A sec. The equivalent electric current is
5
1) 0.4 mA 2) 0.8 mA
x 50 4 l 3) 1.2 mA 4) 1.6 mA
192.  ---- (1) ; 2  100  l  ----- (2)
y 50 3. A current flows in a wire of circular cross
l  50  16.7 section with the free electrons travelling
JG
2 l with drift velocity V . If an equal current
193.   l  40 cm ;
3 100  l flows in a wire of twice the radius, new drift
2 62.5 velocity is
 JG JG
3r 100  62.5 JG V V JG
3 r 1) V 2) 3) 4) 2V
2 4
X l X l 4. An electron of mass 9 x 10-31kg moves
194.  196. 
R 100  l R 100  l around a nucleus in a circular orbit of ra-
l1 L1 dius 2A0 under the action of centripetal
216. l L   217.v = i  l force 3.2N. Then the equivalent electric
l2 L2
current is
 A1  l2  32 3 16 3
218. E ’  n  2m  E 219. r  R   1) 2) 3) 4)
 A2  3 32 3 16
iR RA 5. The current in a conductor varies with time
220. P.G  ; Resistivity   ‘t’ as I = 2- 0.02t amperes. The electric
l L
charge that passes from t = 0 to t = 100
221. v = i  l
sec is
V 1) 50 C 2) 100 C 3) 25 C 4) 75 C
222. Potential gradient =
l
OHM’S LAW AND
 E  E1 l1 IR l COMBINATION OF RESISTANCES
223.  R  r  R  K .l 224. E  ;
l 2 IX
 1
l2
  2

6. Three resistances each of 3  are


V1 l1
225. V  l  V2  0.625V error = 0.65 - 0.625 connected as shown in fig. The resultant
2 2
resistance between A and F is
226. V  i  l

| CURRENT ELECTRTICITY | 203


ACTIVE SITE EDUTECH-9844532971

SrSR||1212THTHCLASS
CLASS|NEET
|NEETPHYSICS
PHYSICS: VOL-1|
: VOL-1|

14. The resultant resistance of two resistors


when connected in series is 48 ohm. The
ratio of their resistances is 3 : 1. The value
of each resistance is
1) 9  2) 2  3) 4  4) 1  1) 20  , 28  2) 32  , 16 
7. Two wires made of same material have 3) 36  , 12  4) 24  , 24 
lengths in the ratio 1 : 2 and their volumes 15. The resistance of a bulb filament is
in the same ratio. The ratio of their 100 at a temperature of 1000 C . If
resistances is its temperature coefficient of
1) 4 : 1 2) 2 : 1 3) 1 : 2 4) 1 : 4 resistance be 0.005 per 0C , its
8. Two wires made of same material have
their electrical resistances in the ratio 1 : r e s i s t a n c e w i l l b e c o m e 200 at
4. If their lengths are in the ratio 1 : 2, the temperature of
ratio of their masses is 1) 3000 C 2) 4000 C
1) 1: 1 2) 1 : 8 3) 8 : 1 4) 2 : 1 3) 5000 C 4) 2000 C
9. There are five equal resistors. 16. In the figure given the value of X resistance
The minimum resistance possible by their will be, when the p.d. between B and D is
combination is 2 ohm. The maximum zero
possible resistance we can make with them
is
1) 25 ohm 2) 50 ohm
3) 100 ohm 4) 150 ohm
10. An electric current is passed through a
circuit containing two wires of the same
material, connected in parallel. If the
lengths and radii of the wires are in the
ratio 4/3 and 2/3, then the ratio of the
currents passing through the wires will be
1) 3 2) 1/3 3) 8/9 4) 2
11. A current of 1 A is passed through two
resistances 1  and 2  connected in
parallel. The current flowing through 2 
resistor will be 1) 4 ohm 2) 6 ohm
1) 1/3 A 2) 1 A 3) 2/3 A 4) 3 A 3) 8 ohm 4) 9 ohm
12. The colour coded resistance of carbon 17. The combined resistance of two conductors
resistance is (Initial three bands are red in series is 1  . If the conductance of one
and fourth band is silver) conductor is 1.1 Siemen, the conductance
1) 222.  10% 2) 2200   10% of the other conductor in Siemen is
1) 10 2) 11 3) 1 4) 1.1
3) 333   5% 4) 33000   10%
13. The resistance of a wire is 10 ohm. The 18. When two resistances are connected in
resistance of a wire whose length is twice parallel then the equivalent resistance is
and the radius is half, if it is made of same 6/5. When one of the resistance is
material is removed then the effective resistance
is 2 . The resistance of the wire
1) 20  2) 5  3) 80  4) 40 
removed will be
204 | CURRENT ELECTRTICITY |
ACTIVE SITE EDUTECH-9844532971

Sr | 12TH CLASS |NEET PHYSICS : VOL-1|

3 6 26. A technician has only two resistance coils.


1) 3 ohm 2) 2 ohm 3) ohm 4) ohm By using them in series or in parallel he is
5 5
able to obtain the resistance 3,4,12 and 16
19. A material ‘B’ has twice the specific resis-
ohms. The resistance of two coils are
tance of ‘A’. A circular wire made of ‘B’
1) 6, 10 2) 4, 12 3) 7, 9 4) 4, 16
has twice the diameter of a wire made of
‘A’. Then for the two wires to have the 27. The effective resistance between A&B in
same resistance, the ratio lB/lA of their the given circuit is
respective lengths must
1) 1 2) 1/2 3) 1/4 4) 2/1
20. If a wire of resistance ‘R’ is melted and
recasted in to half of its length, then the
new resistance of the wire will be
1) R/4 2) R/2 3) R 4) 2R
21. When a wire is drawn until its radius
decreases by 3%. Then percentage of
1) 7  2) 2  3) 6  4) 5 
increase in resistance is
28. The effective resistance between A and B
1) 10% 2) 9% 3) 6% 4) 12%
is 3 then the value of R is
22. When three wires of unequal resistances
are given the number of combinations they
can be made to give different resistances
is
1) 6 2) 4 3) 2 4) 8
23. The resistance of a coil is 4.2 at 1000C
and the temperature coefficient of
resistance of its material is 0.004/0C. Its
resistance at 00C is
1) 6.5  2) 5  3) 3  4) 2.5  1) 2  2) 4  3) 6  4) 8 
24. You are given several identical resistors 29. The effective resistance between A and B
each of value 10 and each capable of in the given circuit is
carrying a maximum current of 1A. It is
required to make a suitable combination
of these to resistances to produce a
resistance of 5 which can carry a current
of 4A. The minimum number of resistors
required for this job is 1) 2  2) 4  3) 3  4) 6 
1) 4 2) 8 3) 10 4) 20
30. Four resistances 10  , 5  ,7  and 3 
25. A wire of resistance 50 is cut into six are connected so that they form the sides
equal parts and they ae bundled together of a rectangle AB, BC, CD and DA
side by side to form a thicker wire. The respectively. Another resistance of 10 
resistance of the bundle is is connected across the diagonal AC. The
18 9 25 25 equivalent resistance between A and B is
1)  2)  3)  4) 
25 12.5 9 18 1) 2  2) 5  3) 7  4) 10 

| CURRENT ELECTRTICITY | 205


ACTIVE SITE EDUTECH-9844532971

SrSR||1212THTHCLASS
CLASS|NEET
|NEETPHYSICS
PHYSICS: VOL-1|
: VOL-1|

31. A 3  resistor and a 6  resistor are 36. The equivalent resistance across XY in fig.
connected in parallel and the combination
is connected in series to a battery of 5V
and a 3  resistor. The potential difference
across the 6  resistor
1) 2V 2) 4V 3) 3V 4) 1V
32. You are given a wire of length 100 cm and
linear resistance of 1 ohm/cm. If it is cut
1) r 2) 2r 3) 4r 4) r / 2
into two parts, so that when they are in
parallel, the effective resistance is 24 ohm. 37. If the resistance of a circuit having 12V
The lengths of the two parts are source is increased by 4  , the current
drops by 0.5A. What is the original resis-
1) 30cm & 70cm 2) 60cm & 40cm tance of circuit
3) 70cm & 30cm 4) 20cm & 80cm
1
1) 4 2) 8 3) 16  4) 
33. The resistance of a platinum wire of a 16
platinum resistance thermometer at the
38. An electric current is passed through a
ice point is 5  and at steam point is
circuit containing two wires of the same
5.4  . When the thermometer is inserted material connected in parallel. If the
in a hot bath, the resistance of the lengths and radii of the wire are in the
platinum wire is 6.2  . Find the ratio 4/3 and 2/3, then the ratio of the
temperature of the hot bath. currents passing through the wires will be
1) 3000o C 2) 30o C 1) 1/3 2) 3/1 3) 4/3 4) 3/4
39. When ‘n’ wires which are identical are
3) 300o C 4) 300 K
connected in series, the effective resistance
34. Three unequal resistors in parallel are exceeds that when they are in parallel by
equivalent to a resistance 1 ohm. If two
X/Y  . Then the resistance of each wire is
of them are in the ratio 1 : 2 and if no
resistance value is fractional, the largest xn yn
of the three resistance in ohm is 1) 2)
y ( n  1)
2
x(n 2  1)
1) 4 2) 6 3) 8 4) 12
35. A carbon filament has resistance of xn yn
3) 4)
120  at 0o C what must be the y (n  1) x(n  1)
resistance of a copper filament
40. The equivalent resistance across A and B is
connected in series with carbon so that
combination has same resistance at all
temperatures

(carbon  5 104 / o C,  copper  4  10 3 / o C )

1) 120  2) 15 

3) 60  4) 210  1) 2  2) 4  3) 8  4) 12 

206 | CURRENT ELECTRTICITY |


ACTIVE SITE EDUTECH-9844532971

Sr | 12TH CLASS |NEET PHYSICS : VOL-1|


41. An ammeter A is connected as shown in the ELECTRIC POWER & JOULE’S LAW
diagram. Ammeter reading is
44. An electric bulb is rated 220 volt and 100
watt. Power consumed by it when operated
on 110 volt is
1) 50 watt 2) 75 watt
3) 90 watt 4) 25 watt
45. A heater coil is cut in to two parts of equal
length and only one of them is used in the
heater. The ratio of the heat produced by
this half-coil to that by the original coil is
1) 2 : 1 2) 1 : 2 3) 1 : 4 4) 4 : 1
46. If the electric current in a lamp decreases
by 5% then the power output decreases
by
E 2E
1) 2) 1) 20% 2) 10% 3) 5% 4) 2.5%
r r
47. Two electric bulbs whose resistances are
r E in the ratio of 1 : 2 are connected in parallel
3) 4) to a constant voltage source. The powers
2E 2r
42. Equivalent resistance between the points dissipated in them have the ratio
A and B (in  ) 1) 1 : 2 2) 1 : 1 3) 2 : 1 4) 1 : 4
48. A bulb rated 60 W -120V is connected to
80V mains. What is the current through the
bulb

1 1 1 1 1 2 5 3
1) 2) 1 3) 2 4) 3 1) A 2) A 3) A 4) A
5 4 3 2 3 3 3 5
43. If a wire is stretched to make it 0.1% 49. An electric bulb has the following specifi-
longer, its resistance will : [Mains-2011] cations 100 watt, 220 volt. The resistance
1) increase by 0.2% of bulb
1) 384  2) 484 
2) decrease by 0.2%
3) 344  4) 584 
3) decrease by 0.05%
50. A 200W and 100W bulbs, both meant for
4) increase by 0.05% operation at 220V, are connected in series
to 220V. The power consumption by the
combination is nearly
1) 2 2) 2 3) 3 4)1 5) 2 6) 4 1) 46 W 2) 67 W 3) 56 W 4) 75 W
7) 4 8) 1 9) 2 10) 2 11) 1 12)2 51. Five bulbs, each rated at 40 W-220 V are
13) 3 14) 3 15) 2 16) 3 17) 2 18) 1 used for 5 hours daily on 20V line. How
19) 4 20) 1 21)4 22) 4 23) 3 24) 2 may units of electric energy is consumed
25) 4 26) 2 27) 3 28) 3 29) 3 30) 2 in a month of 30 days?
31) 1 32) 2 33) 3 34) 2 35) 2 36) 4 1) 20 units 2) 25 units
37) 2 38) 1 39) 1 40) 2 41) 2 42) 3 3) 15 units 4) 30 units
43) 1

| CURRENT ELECTRTICITY | 207


ACTIVE SITE EDUTECH-9844532971

SrSR||1212THTHCLASS
CLASS|NEET
|NEETPHYSICS
PHYSICS: VOL-1|
: VOL-1|
52. An electric Kettle has two heating coils. 58. Two electric bulbs rated 25 W - 220 V and
When one of them is switched on water in 100 W – 220 V are connected in series to a
it boils in 6 minutes and when other is 440 V supply. Which of the bulbs will fuse?
switched on water boils in 4 minutes. In [Mains-2012]
what time will the water boil if both coil are 1) Both 2) 100 W 3) 25 W 4) Neither
switched on simultaneously
1) 1.6 min 2) 2.8 min 3) 2.4 min 4) 3 min
53. A 10 V storage battery of negligible
54) 4 55) 1 56) 2 57) 4
internal resistance is connected across a
58) 3
50  resistor. How much heat energy is
produced in the resistor in 1 hour CELLS AND COMBINATION OF CELLS
1) 7200J 2) 6200J 3) 5200J 4) 4200J
59. A cell of emf 6V is being charged by 1A
current. If the internal resistance of the
cell is 1 ohm, the potential difference across
44) 4 45) 1 46) 2 47) 3 the terminals of the cell is
48) 1 49) 2 50) 2 51) 4
1) 5V 2) 7V 3) 6V 4) 8V
52) 3 53) 1
60. When two identical cells are connected
ELECTRIC POWER either in series or in parallel across 2 ohm
resistor they send the same current through
54. Two resistances R1 and R2 when it. The internal resistance of each cell is
connected in series consume power equal 1) 2 ohm 2) 1.2 ohm
to 25W. When connected in parallel they 3) 12 ohm 4) 21 ohm
consume 100W. The ratio of power of each 61. The emf of a Daniel cell is 1.08V. When
is the terminals of the cells are connected to
1) 1/4 2) 1/3 3) 1/2 4) 1 a resistance of 3  , the potential
55. Two electric bulbs marked 500 W, 220 V difference across the terminals is found to
are put in series with 110V line. The power be 0.6V. Then the internal resistance of the
dissipated in each of the bulb is cell is
125 25 1) 1.8  2) 2.4 
1) W 2) W
4 4
3) 3.24  4) 0.2 
225 325
3) W 4) W 62. Four cells each of emf 2V and internal
4 4
resistance 1 ohm are connected in parallel
56. A conductor of resistance 3 ohm is with an external resistance of 6 ohm. The
stretched uniformly till its length is doubled. current in the external resistance is
The wire now is bent in the form of an
1) 0.32 A 2) 0.16 A 3) 0.2 A 4) 0.6 A
equilateral triangle. The effective
resistance between the ends of any side of 63. A student is asked to connected four cells
the triangle in ohms is of emf of 1 V and internal resistance 0.5
ohm in series with an external resistance
1) 9/2 2) 8/3 3) 2 4) 1
of 1 ohm. But one cell is wrongly connected
57. The energy in kilowatt hour is consumed by him with its terminal reversed, the
in operating ten 50W bulbs for 10 hours current in the circuit is
per day in a month of 30 days is
1) 1500 2) 15000 3) 15 4) 150 1 2 3 4
1) A 2) A 3) A 4) A
3 3 4 3
208 | CURRENT ELECTRTICITY |
ACTIVE SITE EDUTECH-9844532971

Sr | 12TH CLASS |NEET PHYSICS : VOL-1|


64. Two cells of emf 1.25V, 0.75V and each of 71. The potential difference across the terminals
internal resistance 1  are connected in of a battery is 10 V when there is a current
parallel. The effective emf will be of 3A in the battery from the negative to
the positive terminal. When the current is
1) 1 V 2) 1.25 V 3) 2 V 4) 0.5 V 2A in the reverse direction, the potential
65. The emf of a cell is 2V. When the terminals difference becomes 15V. The internal
of the cell is connected to a resistance 4 . resistance of the battery is
The potential difference across the termi-
1) 1 2) 0.4 3) 0.6 4) 0.8
nals, if internal resistance of cell is 1 is
72. Two cells of emf 3V and 5V and internal
3 8 6 5 resistance r1 and r2 respectively are in
1) V 2) V 3) V 4) V
5 5 5 8 series with an external resistance R. If the
66. If the external resistance is equal to internal p.d. across 1st cell is zero, then R is
resistance of a cell of emf E. The current
across the circuit is 5r1  3r2 2r1  3r2
1) 2)
3 4
E r r E
1) 2) 3) 4) 3r1  5r2 4r1  5r2
r E 2E 2r 3) 4)
3 3
67. Two cells each of emf 10V and each 1
internal resistance are used to send a current 73. A battery when connected by a resistance
through a wire of 2 resistance. The cells of 16 gives a terminal voltage of 12V.
are arranged in parallel. Then the current and when connected by a resistance of 10
through the circuit gives a terminal voltage of 11V. Then the
1) 2A 2) 4A 3) 3A 4) 5A emf of the battery and its internal resis-
tance
68. Two batteries of different emf and internal
resistances connected in series with each 1) 12.8 V, 3.85  2) 14.1 V, 2.85 
other and with an external load resistor.
3) 10.7 V,4.85  4) 9 V, 5.85 
The current is 3.0 A. When the polarity of
one battery is reversed, the current 74. When a resistor of 11 ohm is connected in
becomes 1.0 A. The ratio of the emf of the series with an electric cell the current flowing
two batteries is in it is 0.5A. Instead, when a resistor of 5
1) 2.5 : 1 2) 2 : 1 3) 3 : 2 4) 1 : 1 is connected to the same electric cell in
69. The pd across terminals of a cell is found to series, the current increases by 0.4A. The
be 29 volt and 28 volt respectively when it internal resistance of cell is
delivers a current of 1 ampere and 2 1) 1.5  2) 2  3) 2.5  4) 6 
ampere respectively. The emf and internal
resistance of a cell are respectively 75. Two cells of emf 4V and 8V are connected
1)30V, 2  2) 30V,1  to two resister 4 and 6 as shown. If
8V cell is short circuited. Then current
3) 29V, 1  4) 28V,2 
through resistance 4 and 6
70. The current in a circuit containing a battery
connected to 2  resistance is 0.9 A. When
a resistance of 7  connected to the same
battery, the current observed in the circuit
is 0.3A. Then the internal resistance of the
battery is
1) 0.1  2) 0.5  3) 1  4) Zero 1) 2 A 2) 1 A 3) 2.5 A 4) 3 A

| CURRENT ELECTRTICITY | 209


ACTIVE SITE EDUTECH-9844532971

SrSR||1212THTHCLASS
CLASS|NEET
|NEETPHYSICS
PHYSICS: VOL-1|
: VOL-1|
76. If in the circuit shown below, the internal 80. A battery of emf ‘E’ and internal resistance
resistance of the battery is 1 andVp and ‘r’ is connected to a resistor of resistance
VQ are the potentials at P and Q respec- ‘ r1 ’ and Q Joules of heat is produced in a
tively, the potential difference between the
certain time ‘t’. When the same battery is
points P and Q is
connected to another resistor of resistance
r2 the same quantity of heat is produced in
the same time ‘t’. Then, the value of ‘r’ is
(M-2011)
r12 r22 1
1) 2) 3) (r1  r2 ) 4) r1 r2
r2 r1 2
1) 9 V 2) 11 V 3) 7 V 4) 6 V 81. The emf of a cell E is 15 V as shown in the
77. Voltmeter reading in the given circuit is figure with an internal resistance of 0.5  .
(voltmeter is ideal) Then the value of the current drawn from
the cell is (M-2013)

1) 6 V 2) 8 V 3) 10 V 4) 14 V
78. For a cell the graph between the p.d(v) 1) 1 A 2) 3 A 3) 2 A 4) 5 A
across the terminals of the cells and the
current (I) drawn from the cell as shown.
The emf and internal resistance is
59) 2 60) 1 61) 2 62) 1
63) 2 64) 1 65) 2 66) 4
67) 2 68) 2 69) 2 70) 2
71) 1 72) 1 73) 2 74) 3
75) 2 76) 1 77) 2 78) 4
79) 2 80) 4 81) 1

KIRCHOFF’S LAWS, WHEATSTONE BRIDGE


82. The figure below shows current in a part
3 1 2 of electric circuit. The current i is
1)  2)  3) 3 4) 
2 3 3
79 The minimum number of cells in mixed
grouping required to produced a maximum
current of 1A through external resistance
of 20 given the emf of each cell is 2V
and internal resistance 1 is
1) 25 2) 20 3) 16 4) 30 1) 1.7amp 2) 3.7 amp
3)1.3 amp 4) 1 amp
210 | CURRENT ELECTRTICITY |
ACTIVE SITE EDUTECH-9844532971

Sr | 12TH CLASS |NEET PHYSICS : VOL-1|

83. Current in the main circuit shown is 88. The current i drawn from the 5 volt source
will be

1) 1.5 A 2) 2 A 3) 0.6 A 4) 1 A
84. Find ‘i’ for the given loop. 1) 0.5 2) 2 A 3) 1.5 A 4) 3 A
89. In the given circuit which is a part of a
closed circuit the current i 1, i 2 are
respcetively

6 8 1
1) A 2) A 3) A 4) 1A
5 9 2
85. The potential difference between points A 1) 0.4 A 2) 0.6 A
and B is
3) 1.6 A 4) 2 A
90. A 5V battery with internal resistance 2 
1) 0 V 2) 10 V 3) 4 V 4) 5 V and a 2V battery with internal resistance
86. In wheat stone bridge, P and Q are approxi-
1  are connected to a 10  resistor as
mately equal. When R is 500 , the bridge shown in the figure [Mains-2008]
is balanced. On interchanging P and Q, the
values of R is 505 for balancing . The
value of ‘S’ is
1) 500.5  2) 501.5 
3) 502.5  4) 503.5 
87. To balance the bridge in the circuit, the val-
ues of R is

The current in the 10  resistor is


1) 0.27 A P2 to P1 2) 0.03 A P1 to P2
1) 8  2) 4  3) 20  4) 12  3) 0.03 A P2 to P1 4) 0.27 A P1 to P2
| CURRENT ELECTRTICITY | 211
ACTIVE SITE EDUTECH-9844532971

SrSR||1212THTHCLASS
CLASS|NEET
|NEETPHYSICS
PHYSICS: VOL-1|
: VOL-1|
96. When a conductor is connected in the left
gap and known resistance in the right gap
the balancing length is 50cm. If the wire is
82) 1 83) 4 84) 2 85) 1 stretched so that its length increased by
86) 3 87) 1 88) 1 89) 2
90) 3 20% , New balancing length is
1)40.98cm 2)38.23cm 3)42.56cm 4)48.21cm
METRE BRIDGE 97. In a meter bridge experiment when a
resistance wire is connected in the left gap,
91. The point in a Meter bridge is at 35.6 cm. If the balance point is found at the 30th cm.
the resistances in the gaps are When the wire is replaced by another wire,
interchanged,the new balance point is the balance point is found at the 60th cm.
1) 64.4 cm 2) 56 cm find the balance point when the two wires
3) 41.2 cm 4) 56.7 cm connected in series and in parallel in the
left gap successively
92. In a meter bridge expt, when the
1) 55,20 cm 2) 65.8,25 cm
resistances in the gaps are interchanged
the balance point is increases by 10cm. The 3) 70,23 cm 4) 80,25 cm
ratio of the resistances is 98. Shown in the figure below is a meter-bridge
set up with null deflection in the
15 12 11 10 galvanometer.
1) 2) 3) 4)
5 8 9 9
93. When an unknown resistance and a
resistance 6 are connected in the left and
right gaps of a meter bridge, the balance
point is obtained at 50cm. If 3 resistance
is connected in parallel to resistance in right
gap, the balance point is
1) decrease by 25 cm 2) increase by 25 cm
3) decrease by 16.7 cm
4) increase by 16.7 cm
The value of the unknown resistor R is
94 When un known resistance and a resistance
[Mains-2008]
of 5 are used in left and right gaps of
1) 13.75  2) 220  3) 110  4) 55 
meter bridge the balancing point is 50cm.
The balancing point if 5 resistance is
now connected in series to the resistor in
91) 1 92) 3 93) 2 94) 2
right gap 95) 3 96) 1 97) 2 98) 2
1) 20 cm 2) 33.3 cm 3) 60 cm 4) 60 cm
95. In a meter bridge experiment two unknown POTENTIOMETER
resistances X and Y are connected to left 99. In a potentiometer the balance length with
and right gaps of a meter bridge and the standard cadmium cell is 509 cm. The emf
balancing point is obtained at 20cm from of a cell which when connected in the place
right (X>Y) the new position of the null point of the standard cell gave a balance length
from left if one decides balance a of 750 cm is (emf of standard cell is
resistance of 4X against Y. 1.018V)
1) 114 cm 2) 80 cm 3) 94.1 cm 4) 70 cm 1) 1.5V 2) 0.5V 3) 1.08V 4) 1.2V

212 | CURRENT ELECTRTICITY |


ACTIVE SITE EDUTECH-9844532971

Sr | 12TH CLASS |NEET PHYSICS : VOL-1|


100. Two cells of emf’s E1 and E2 when placed 106. The ratio of potential gradients is 1 :
in series produce null deflection at a 2, the resistance of two potentiometer
distance of 204 cm in a potentiometer. When wires of same length are 2  & 4
one cell is reversed they produce null respectively. The current flowing
deflection at 36 cm if E1 1.4v then E2= through them are in the ratio
1) 0.98 V 2) 2.47 V 1) 1 : 2 2) 2 : 1 3) 1 : 3 4) 1 : 1
3) 0.098 V 4) 98.8 V
107.The length of potentiometer wire is
101. When 6 identical cells of no internal 100 cm and the emf of its standard cell
resistance are connected in series in the is E volts. It is employed to measure
secondary circuit of a poetntiometer, the the emf of a battery whose internal
balancing length is l. If two of them are resistance is 0.5 ohm. If the balance
wrongly connected the balancing length point is obtained at l = 30 cm from
becomes positive end, the emf of the battery is
l l 2l 30 E 30 E
1) 2) 3) l 4) 1) 2)
4 3 3 100.5 100  0.5
102. In an experiment to determine the internal
resistance of a cell with potentiometer, the 30 E 100 E
3) 4)
balancing length is 165cm. When a 100 30
resistance of 5 ohm is joined in parallel with 108.1 ohm resistance is in series with an
the cell the balancing length is 150cm. The Ammeter which is balanced by 75 cm
internal resistance of cell is of potentiometer wire. A standard cell
1) 2.2  2) 1.1  of 1.02 V is balanced by 50 cm. The
3) 3.3  4) 0.5  ammeter shows a reading of 1.5 A. The
103. The resistivity of a potentiometer wire is error in the ammeter reading is
40 x 10-8   m and its area of cross section 1) 0.002 A 2) 0.03 A
is 8 x 10-6 m2. If 0.2A current is flowing 3) 1.01 A 4) no error
through the wire, the potential gradient will be
1) 10–2 V/m 2) 10–1 V/m
3) 3.2 × 10 V/m
–2
4) 1 V/m 99) 1 100) 1 101) 2 102) 4
103) 1 104) 3 105) 1 106) 4
104. The emf of a cell is Ev, and its its internal 107) 3 108) 2
resistance is 1 . A resistance of 4 is
joined to battery in parallel. This is
connected in secondary circuit of potentiometer.
The balancing length is 160cm. If 1V cell
balances for 100cm of potentio meter wire,
HINTS SOLUTIONS
the emf of cell E is
1) 1 V 2) 3 V 3) 2 V 4) 4 V ne it
1. i n 2. i  q f
105. A potentiometer wire 10 m long has a t e
resistance of 40 . It is connected in series 1 q qv  mv 2 
Vd  i   ' F 
with a resistance box and a 2 v storage cell. 3.
r2
4. t 2  r ;  r 
If the potential gradient along the wire is
t2
0.01V/m the resistance unplugged in the
box is 5. q   idt
t1
1) 760 2) 260 3) 1060 4) 960 6. The 3 resistances are parallel

| CURRENT ELECTRTICITY | 213


ACTIVE SITE EDUTECH-9844532971

SrSR||1212THTHCLASS
CLASS|NEET
|NEETPHYSICS
PHYSICS: VOL-1|
: VOL-1|
27. Using combination of resistors
l2 28. Using combination of resistors
7. R  l2 ' V constant 8. R 
m 29. Using combination of resistors
R R 30. Combination of resistors
9. 2 Rmax  5 R Rmin  31. Combination of resistors
5 5
32. Combination of resistors
i1 r12 l2 iR1
10.   11 . i2  R  R 33. R2  R1 1  T 
i2 r22 l1 1 2
1 1 1
R1 l1 r 2 34.    1 ; R2 : R3  1: 2
  2 R1 R2 R3
12. R  22  10 2  10% 13. 2
R2 l2 r 1 35. R11  R2 2
R1 R2 36. Combination of resistors
14. RS  R1  R2 , RP  Solving for R1 & R2 37. V = ( i – 0.5) (R + 4 )
R1  R2 38. V - iR V = constant
i1 R2 l
R2  R1 i1 R1  i2 R2 ; i  R ; ' R 
15.  A
R1t2  R2t1 2 1

x R
16. Balanced Wheatstone’s bridge 39. nR  
y n
 8X  40. Combination of resistors
  3  8X  5E 3E
(6  15)  8  X   3   --------
  41. i=
 6  4  ; 21   8  X  r r
 15    4   18 6
 2  2
42.
8X
 4  X  8
8 X
17 R1 + R2 = 1 ohm.
RR 6
18. R  R  5 . If R is removed R  2
1 2

2 1
1 2

2R 6
  5 R  6  3R  R  3
2

2 R 5
2 2 2
1 7 1
2 Req  1   1   2 
3 3 3
A RA r2
19. R  A l  V 2

A   43. R  l 2 44. P 
1 R
20. R  A 2 21. R  4 V 2
Q R
r 45. W  JQ  Q   1  2
22. Number of combinations=2n RJ Q2 R1
P I
23. Rt  R0 1   t  46. P  i 2 R  P i 2   100  2   100
P I
R V 2
V’ 2
V
24. RP  ; i p  mi ;  m × n = ............... 47. P  48. R  and i 
n R P’ R
R RS  Rs2  4 Rs Rp V 2
PP
25. R ’  26. R1  49. R  50. P  1 2
n2 2 P P1  P2
51. no. of watts  no. of hours
Rs  Rs2  4 Rs Rp
R2  1000
2

214 | CURRENT ELECTRTICITY |


ACTIVE SITE EDUTECH-9844532971

Sr | 12TH CLASS |NEET PHYSICS : VOL-1|


tt1 2 E
52. ts  t1  t2 ; tp  77. V = iR 78. i =
t1 t2 r
79. No. of cells = m × n
V 2
imax = mR = nr
53. Q  t  Q  7200J
R V2 E 2 r1 V2 E 2 .r2
79. Q   ; Q  
PP r1 (r  r1 ) 2 r2 (r  r2 ) 2
54. Ps  1 2 ; PP  P1  P2
P1  P2 80. Q1  Q2 ; i12 rt
1  i2 r2t
2

V2 V 81 using Wheastone’s bridge


55. R1  R2  R ; i ;  P  i2 R
P 2R 82. K.C.L ; 83. K.C.L ; 84. K.C.L;
R1 R2 P R P R
56. R  L , Reff  R  R
2
85. K.C.L; 86.  ; 87. 
1 2 Q S Q S
57. P  i 2 58. P  Vi 59 . V  E  ir
88 using Kirchhoff’s laws
nE E  E V 
60. iS  iP ;  61. r  R 89. Applying Kirchhoff’s law
 R  nr  R r 
  V 
90. Applying Kirchhoff’s law
 
 n
E 2 2 4 8 P 35.6 Q 64.4 X l
62. i     91.  ,  92. 
r 1 25 25 Q 64.4 R 35.6 R 100  l
R 6
n 4
X l X l
63. i
 N  2 n  E  4  2   1  2  1 93.  95. 
R 100  l Y 100  l
R  Nr 1  4  0.5 3
E1r2  E2 r1 X l X l
64. Eeff  65. V  E  i r 96.  97. 
r1  r2 R 100  l R 100  l
E
66. i 67. i  E x l1 55 20 E1 l1
Rr r 98. y  l  R  80 99. E  l
R 2 2 2
n
E  E2 E1 A 1  A 2
68. 3 1      1
r1  r2  R 100. E  A  A
2 1 2
E1  E2
     2  ;
1
1
r1  r2  R 2 
101. N E  l1 ,  N  2m  E l2
 E V  A l  i
69. r   R 102. r  R  1 2  103. P. G  ;
 V   A2  A
E
70.  0.9      1
2r  A1  l2  E R
E 104. r  R   105. K 
 0.3      2   A2  R  Rs  L
7r
71. E  3r  10 ... i ; E  2r  15 ... ii  i1 V1 R2
 E ’ 30
106. 107. V  l , 
1 7  r i2 V2 R1 E 100
solving (i) & (ii) r = 1; 2 2  r  3
  V1 l1 75
V 108. V  l  50  V1  1.53V
72. V = E – ir 73. E   R  r   constant 2 2
R
V V1
74. E = i(R + r) 75.V = iR 76. i = I1   1.53 A , error = 1.53-1.50=0.03
R 1

| CURRENT ELECTRTICITY | 215


ACTIVE SITE EDUTECH-9844532971

SrSR||1212THTHCLASS
CLASS|NEET
|NEETPHYSICS
PHYSICS: VOL-1|
: VOL-1|

G 4. A carbon resistor of (47 ± 4.7) k  is to be


marked with rings of different colours for

WORK AT HOME¨
its identification. The colour code sequence
will be [NEET 2018]
1) Yellow – Green – Violet – Gold
PREVIOUS EXAMS QUESTIONS 2) Yellow – Violet – Orange – Silver
1. Six similar bulbs are connected as shown 3) Violet – Yellow – Orange – Silver
in the figure with a DC source of emf E 4) Green – Orange – Violet – Gold
and zero internal resistance. 5. A set of ‘n’ equal resistors, of value ‘R’
[NEET 2019] each, are connected in series to a battery
of emf ‘E’ and internal resistance ‘R’. The
The ratio of power consumption by the current drawn is I. Now, the ‘n’ resistors
bulbs when (i) all are glowing and (ii) in the are connected in parallel to the same
situation when two from section A and one battery. Then the current drawn from
from section B are glowing, will be : battery becomes 10 I. The value of ‘n’ is
[NEET 2018]
A B
1) 20 2) 11 3) 10 4) 9
6. A battery consists of a variable number ‘n’
of identical cells (having internal resistance
‘r’ each) which are connected in series. The
terminals of the battery are short-circuited
E and the current I is measured. Which of
the graphs shows the correct relationship
2. Which of the following acts as a circuit between I and n?
protecting device? [NEET 2019] [NEET 2018]
1) Conductor 2) Inductor
3) Switch 4) Fuse
3. In the circuits shown below, the readings 1) 2)
of voltmeters and the ammeters will be
10  i1

V1
A1

[NEET 2019]
10 V
3) 4)
C ir c u it 1

7. A potentiometer is an accurate and


10  i2 versatile device to make electrical
10  measurements of E.M.F, because the
V2 A2
method involves: [NEET 2017]
1) Cells
10 V 2) Potential gradients
Circuit 2
3) A condition of no current flow through the
galvanometer
1) V2  V1 and i1  i2 2) V1  V2 and i1  i2
4) A combination of cells, galvanometer and
3) V1  V2 and i1  i2 4) V2  V1 and i1  i2 resistances

216 | CURRENT ELECTRTICITY |


ACTIVE SITE EDUTECH-9844532971

Sr | 12TH CLASS |NEET PHYSICS : VOL-1|


13. A potentiometer wire has length 4 m and
8. The resistance of a wire is ‘R’ ohm. If it is
melted and stretched to ‘n’ times its resistance 8  . The resistance that must be
original length, its new resistance will be connected in series with the wire and an
[NEET 2017] accumulator of e.m.f. 2V, so as to get a
potential gradient 1 mV per cm on the wire is
R R
1) nR 2) 3) n 2 R 4) [AIPMT-2015]
n n2
9. A potentiometer wire is 100 cm long and a 1) 48  2) 32  3) 40  4) 44 
constant potential difference is maintained 14. Two cities are 150 km apart. Electric power
across it. Two cells are connected in series is sent from one city to another city through
first to support one another and then in copper wires. The fall of potential per km is
opposite direction. The balance points are 8 volt and the average resistance per km is
obtained at 50 cm and 10 cm from the 0.5W the power loss in the wires is:
positive end of the wire in the two cases. [AIPMT 2014]
The ratio of emf’s is: [NEET 2016] 1) 19.2 W 2) 19.2 kW
1) 5 : 1 2) 5 : 4 3) 3 : 4 4) 3 : 2 3) 19.2 J 4) 12.2 kW
10. The potential difference 15. The resistance in the two arms of the meter
bridge are 5  and R  , respectively.
VA VB  between the points A and B in When the resistance R is shunted with an
the given figure is: [NEET 2016] equal resistance, the new balance point is
at 1.6 A 1 . The resistance ‘R’ is:
[AIPMT 2014]
1) -3 V 2) +3 V 3) +6V 4) +9 V
11. A circuit contains an ammeter, a battery of
30 V and a resistance of 40 ohm all
connected in series. If the ammeter has a
coil of resistance 40 ohm and a shunt of 20
A1 100 A1
ohm, the reading in the ammeter will
[Re-AIPMT-2015] 1) 10 2) 15 3) 20 4) 25
1) 1A 2) 0.5 A 3) 0.2 A 4) none 16. A potentiometer circuit has been set up for
12. A, B and C are voltmeters of resistance R, finding the internal resistance of a given
1.5R and 3R respectively as shown in the cell. the main battery, used across the
figure. When some potential difference is potentiometer wire, has an emf of 2.0 V and
applied between X and Y, the voltmeter a negligible internal resistance. The
readings are VA, VB and VC respectively, potentiometer wire itself is 4m long, when
the resistance R, connected across the
then [AIPMT-2015]
given cell, has values of infinity and 9.5W.
The balancing lengths on the potentiometer
wire are found to be 3m and 2.85 m,
respectively. The value of internal
1) VA  VB  VC 2) VA  VB  VC resistance of the cell is [AIPMT 2014]

3) VA  VB  VC 4) VA  VB  VC 1) 0.25 2) 0.95 3) 0.5 4) 0.75

| CURRENT ELECTRTICITY | 217


ACTIVE SITE EDUTECH-9844532971

SrSR||1212THTHCLASS
CLASS|NEET
|NEETPHYSICS
PHYSICS: VOL-1|
: VOL-1|
17. The internal resistance of a 2.1 V cell which A1
gives a current of 0.2 A through a resistance
of 10W [NEET-UG 2013]
1) 1.0 2) 2 3) 4 4) none
18. The resistances of the four arms P, Q, R A2
and S in a Wheatstone’s bridge are 10 ohm,
30 ohm, 30 ohm and 90 ohm, respectively. A1 3 A1 1
1) A  8 2) A  2
The e.m.f. and internal resistance of the cell 2 2

are 7 volt and 5 ohm respectively. If the gal- A1 5 A1 1


3)  4) A  3
vanometer resistance it 50 ohm, the current A2 8 2
drawn from the cell will be 22. In the circuit shown the cells A and B have
[NEET-UG 2013] negligible resistance. For VA = 12V, R1 =
1) 2.0 A 2) 1.0 A 3) 0.2 A 4) 0.1 A 500W and R  100 the galvanometer (G)
19. In the potentiometer circuit shown in the shows no deflection. The value of VB is:
figure, the balance length AJ = 60 cm when [AIPMT (Pre) 2012]
switch S is open. When switch S is closed
and the value of R = 5W, the balance length
AJ = 50 cm. The internal resistance of the
cell C’ is [AMU 2013]

1) 12V 2) 6V 3) 4V 4) 2V
23. The power dissipated in the circuit shown
in the figure is 30 Watts. The value of R
is: [AIPMT (Mains) 2012]
1) 1.2  2) 1.0  3) 0.8  4) 0.6 
20. A circuit consisting of five resistors each of
resistance R, forming a Wheatstone’s
bridge. What is the equivalent resistance
of the circuit? [AIIMS 2013]
1) 2R 2) R 3) 2R/3 4) R/2
21. A ring is made of a wire having a resistance
R 0  12 .Find the points A and B as shown 1) 10 2) 30 3) 20 4) 15
in the figure at which a current carrying 24. A cell having an emf  and internal resistance
conductor should be connected so that the r is connected across a variable external
resistance R of the sub circuit between resistance R. As the resistance R is increased,
8 the plot of potential difference V across R is
these points is equal to 
3 given by: [AIPMT (Mains) 2012]
[AIPMT (Pre) 2012]

218 | CURRENT ELECTRTICITY |


ACTIVE SITE EDUTECH-9844532971

Sr | 12TH CLASS |NEET PHYSICS : VOL-1|


30. If power dissipated in the 9  resistor in
the circuit shown in 36 Watt, the potential
difference across the 2  resistor is:
1) 2) [AIPMT (Pre) 2011]

3) 4)
1) 4 Volt 2) 8 Volt 3) 10 Volt 4) 2 Volt
25. The value of current i for the given circuit 31. In the circuit shown in the figure, if the
is [AIIMS 2012] potential at point A is taken to be zero, the
potential at point B is:
[AIPMT (Mains) 2011]

1) +1V 2) –1V 3) +2V 4) –2V


1) 10A 2) 5A 3) 2.5A 4) 20A
32. A potentiometer circuit is set up a shown.
26. Calculate the energy emitted by a bulb of
the potential gradient, across the
100W in 1 min: [AIIMS 2012]
potentiometer wire is k volt/cm and the
1) 100J 2) 10003 J 3) 600 J 4) 6000 J
ammeter, present in the circuit reads 1.0 A
27. If voltage across a bulb rated 220 V - 100 when two way key is switched off. The
W drops by 2.5% of its rated value, the balance points, when the key between the
percentage of the rated value by which the terminals (i) 1 and 2 (ii) 1 and 3, is plugged
power would decrease is
in, are found to be at lengths A 1 cm and
[AIPMT 2012]
1) 5% 2) 10% 3) 20% 4) 2.5% A 2 cm respectively. The magnitudes, of the
28. A wire of resistance 4W is stretched to twice resistors R and X, in ohms, are then, equal
original length. The resistance of stretched respectively, to: [AIPMT (Pre) 2010]
wire would be: [NEET-UG 2012]
1) 16 2) 2 3) 4 4) 8
29. A current of 2A flows through a 2W
resistor when connected across a battery.
The same battery supplies a current of
0.5A when connected across a 9W resis-
tor. The internal resistance of the battery
[AIPMT (Pre) 2011]
1) 0.5W 2) 1/3W 3) 1/4W 4) 1W

| CURRENT ELECTRTICITY | 219


ACTIVE SITE EDUTECH-9844532971

SrSR||1212THTHCLASS
CLASS|NEET
|NEETPHYSICS
PHYSICS: VOL-1|
: VOL-1|
1) kA 1 and kA 2
36. See the electrical circuit shown in this figure.
2) k(A 2  A 1 ) and kA 2 Which of the following equations is a correct
3) kA 1 and k(A 2  A 1 ) equation for it? [AIPMT 2009]
4) k(A 2  A 1 ) and kA 1 R

33. Consider the following two statements: i1 1 r2


(A) Kirchhoff’s junction law follows from the
i2
conservation of charge
r2 2
(B) Kirchhoff’s loop law follows from the
conservation of energy 1) 1  (i1  i 2 )R  i1r1  0
Which of the following is correct? 2) 1  (i1  i 2 )R  i1r1  0
[AIPMT (Pre) 2010] 3)  2  i1r2  1  i1r1  0
1) both (A) and (B) are correct
2) both (A) and (B) are wrong 4)  2  (i1  i 2 )R  i 2 r2  0
3) (A) is correct and (B) is wrong 37. A student measures the terminal potential
4) (A) is wrong and (B) is correct difference (V) of a cell (of emf  and
34. For the current loops shown in the figure, terminal resistance r) as a function of the
Kirchhoff’s loop rule for the loops AHDCBA current (I) flowing through it. The slope, and
and AHDEFGA yields these equations intercept, of the graph between V and I, then
respectively [AMU 2010] respectively, equal: [AIPMT 2009]
(1)  and r (2)  and –r
(3) –r and  (4) r and – 
38. In the circuit shown, the current through the
4  resistor is 1 amp when the points P and
M are connected to a d.c. voltage source.
The potential difference between the points
M and N is: [AIPMT 2008]
1) 30I1 41I3  45  0 &  30I1  21I2  80  0
2) 30I1  41I3  45  0 & 30I1  21I2  80  0
3) 30I1  41I3  45  0 &  30I1  21I2  80  0
4) 30I1  41I3  45  0 &  30I1  21I2  80  0
35. A wire of resistance 12 ohms per meter is 1)0.5 volt 2) 3.2 volt
bent to form a complete circle of radius 10 3) 1.5 volt 4) 1.0 volt
cm. The resistance between its two 39. A current of 3 ampere flows through the 2-
W resistor shown in the circuit. The power
diametrically opposite points, A and B as
dissipated in the 5W resistor is:
shown in the figure, is: [AIPMT 2009]
[AIPMT 2008]
2
A B 4

1 5

1) 6 2) 0.6 3) 3 4) 6 1) 1 watt 2) 5 watt 3) 4 watt 4) 2 watt

220 | CURRENT ELECTRTICITY |


ACTIVE SITE EDUTECH-9844532971

Sr | 12TH CLASS |NEET PHYSICS : VOL-1|


40. Four wires of the same diameter are 44. Two cells, having the same emf, are
connected, in turn, between two points connected in series through an external
maintained at a constant potential resistance R. Cells have internal resistances
difference. Their resistivities and lengths r1 and r2 (r1 > r2) respectively. When the
L are ; and L (wire 1), 1.2 and 1.2 L circuit is closed, the potential difference
(wire 2), 0.9 and 0.9 L (wire 3), and  and across the first cell is zero, the value of R is
1.5 L(wire 4). Rank the wires according to [AIPMT (Prelims)2006]
the rates at which energy is dissipated as r1  r2 r1  r2
heat, greatest first, [AMU 2007] 1) r 1 – r2 2) 3) 4) r1 + r2
2 2
1) 4 > 3 > 1 > 2 2) 4 > 2 > 1 > 3 45. Power dissipated across the 8 resistor in
3) 1 > 2 > 3 > 4 4) 3 > 1 > 2 > 4 the circuit shown here is 2W. The power
41. The total power dissipated in watt in the dissipated in watt units across the 3
circuit shown here is resistor is [AIPMT(Prel)2006]
[AIPMT (Prelims)2007]

1) 4 2) 16 3) 40 4) 54
1) 2 2) 1 3) 0.5 4) 3
42. Three resistances P, Q, R each of 2 and
46. Kirchhoff’s first and second laws for
an unknown resistance S form the four arms
electrical circuits are consequences of
of a Wheatstone’s bridge circuit. When a [AIPMT (Prelims)2006]
resistance of 6 is connected in parallel to 1) conservation of energy
S the bridge gets balanced. What is the 2) conservation of electric charge and energy
value of S? [AIPMT (Prelims)2007] respectively
1) 1 2) 2 3) 3 4) 6 3) conservation of electric charge
4) conservation of energy and electric charge
43. In the circuit shown, if a conducting wire is
respectively
connected between points A and B, the
47. When a wire of uniform cross-section a,
current in this wire will: [AIPMT 2006] length  and resistance R is bent into a
A complete circle, resistance between any two
4 4 of diametrically opposite points will be:
[AIPMT 2005]
R R R
3
1) 2) 3) 4) 4R
1 2 4 8
B 48. In potentiometer experiment when
V terminals of the cell is at distance of 52 cm,
1) flow from A to B then no current flows through it. When 5
shunt resistance is connected in it then
2) flow in the direction which will be decided balance length is at 40 cm. The internal
by the value of V resistance of the cell is [RPMT 2005]
3) be zero 200 52
1) 5  2)  3) 4) 1.5 
4) flow from B to A 52 8 

| CURRENT ELECTRTICITY | 221


ACTIVE SITE EDUTECH-9844532971

SrSR||1212THTHCLASS
CLASS|NEET
|NEETPHYSICS
PHYSICS: VOL-1|
: VOL-1|

49. A potentiometer wire has resistance 40


and its length is 10m. It is connected by a
resistance of 760 in series. If emf of 1) 2 2) 4 3) 3 4) 2 5) 3
battery is 2V then potential gradient is: 6) 3 7) 3 8) 3 9) 4 10) 4
[RPMT 2005] 11) 4 12) 2 13) 2 14) 2 15) 2
1) 0.5 106 V / m 2) 1106 V / m 16) 3 17) 4 18) 3 19) 2 20) 2
21) 2 22) 4 23) 1 24) 1 25) 2
3) 1102 V / m 4) 2 106 V / m 26) 4 27) 1 28) 1 29) 4 30) 3
31) 1 32) 3 33) 1 34) 4 35) 2
50. A 5A fuse wire can withstand a maximum 36) 2 37) 3 38) 2 39) 2 40) 4
power of 1 W in circuit. The resistance of 41) 4 42) 3 43) 4 44) 1 45) 4
the fuse wire is [AIPMT (Prelims)2005] 46) 2 47) 2 48) 4 49) 3 50) 4
1) 0.2 2) 5 3) 0.4 4) 0.04 51) 2 52) 3
51. For the network shown in the figure, the
value of the current i is
[AIPMT (Prelims)2005] HINTS SOLUTIONS
2 R R 2R
1. (i) All bulbs are glowing Req   
3 3 3
4 4 3 R R

6 R R
i R R
V
E
9V 5V 5V 18V E 2 3E 2
1) 2) 3) 4) Power  Pi    ...(1)
35 18 9 5 Req 2 R
52. Two batteries, one of emf 18 V and internal (ii) Two from section A and one from section
B are glowing.
resistance 2 and the other of emf 12 V
and internal resistance 1, are connected R
as shown. The voltmeter V will record a R

reading of [AIPMT (Prelims)2005]


R

R 3R
Req  R
2 2
2E2
Power  Pf   ...(2)
3R
2
Pi 3E 3R
1) 15V 2) 30 V 3) 14V 4) 18 V   9:4
Pf 2 R 2 E 2

222 | CURRENT ELECTRTICITY |


ACTIVE SITE EDUTECH-9844532971

Sr | 12TH CLASS |NEET PHYSICS : VOL-1|


2. Fuse wire has less melting point so when excess 12. Let ‘V’ be the potential difference between
current flows, due to heat produced in it, it x and y
melts. V V V
3. For ideal voltmeter, resistance is infinite and for i   0.5
the ideal ammeter, resistance is zero. R eq 2R R
10 0.5V
V1  i1  10   10  10 volt VA   R  0.5V
10 R
10 0.5V
V2  i2  10   10  10 volt VC   R  0.5V ; VA  VB  VC
10 R
10 V 2
V1  V2 ; i1  i2  1A 13. i
10  8 R
4.  47  4.7  k  47 103  10% V 4 40
K  AB  Volt / m  mv / cm
 Yellow – Violet – Orange – Silver L 8 R 8R
E R  32
5. I ...(i)
nR  R 14. R = 75 ohm ;V = 1200V; P = 19.2 KW
E ...(ii) 5 l1 10 1.6 1
10 I  15.  ...(i); 
R
R R 100  l1 R 100  1.6 1 ....(ii)
n by equation (i) & (ii);  1  25cm ; R  15
Dividing (ii) by (i), 10  
n  1 R
   
1  16. r   1 2 R
  1 R  2 
n 
After solving the equation, n = 10 17. V  E  ir ; 2  2.1  (.2)r
n  0.2r  .1 ; r  0.5
6. I 
nr r V 7
So, I is independent of n and I is constant. 18. R eq  35 ; i    .2A
R 35
   
19. r   1 2  R = 1 ohm
 2 
20. Req = R
21. R 2  R1  12 ....(i)
7. Reading of potentiometer is accurate because
during taking reading it does not draw any R 1R 2 RR 8
 1 2 
current from the circuit. R1  R 2 12 3

R2 I 22 n 2 I12 R2  n 2 R 2  R 1  4 ....(ii)
8.  2 2 ; R ; R2  n 2 R1 by equation (i) and (ii)
R1 I1 I1 1

R1  8 R1 l1 1
3 E1 ;  
9. E1  E2   50 , E1  E2  10 : 2  E R 2  4 R 2 l2 2
2

10. VA  VB   2  2   3   2 1 = 9v 1
22. Current in R  A
160 30  3 9 50
11. R eq   ;i   ; V  22.5V 1
3 160 16 P.D. across R is V  iR   100
7.5 3 50
V1  7.5V ; i1  
VB  2V
400 16

| CURRENT ELECTRTICITY | 223


ACTIVE SITE EDUTECH-9844532971

SrSR||1212THTHCLASS
CLASS|NEET
|NEETPHYSICS
PHYSICS: VOL-1|
: VOL-1|
V 10 1 1 1
23. i   2amp 35.  
R 5 R 12r 12r
30 R  6r  0.6 
P  vi ; i  3A ;
10 36. By Kirchhoff second law
V 1  (i1  i 2 )R  i1r1  0
R  10ohm
I
dV
 E  dV Er 37.  r
24. V  IR    R ; dR  (r  R)2 di
rR
 slope decreases on increasing R E
5
25. R  2.5 V
2
V 25 i
i   10A ;  5A i
R 2.5 2
intercept = E
Current i  10  5  5A
26. E = Pt = 100 × 60 = 6000 J 38. Vnet  4V
P  V  4 16
27. 100 2 100  2(2.5)  5% R lower  1.25 ; i lower   A
P V  1.25 5
2
28. R1  n 2 R  (2)  4  16 16
VMN   1  3.2V
29. 2r  4  E ....(i) 5
0.5r  4.5  E ...(ii) 39. Net voltage 6 volt

1 V2 2
by equation (i) and (ii) r i  1amp flows in 5ohm P  i r
3 R
40. R 3  R1  R 2  R 4
V2
30. For 9 36   V  18 volt
9 V2
H ; 3 1 2  4
current in 9  2A R
current in 6  3A 41. Net resistance R = 6 ohm
P.D. across 2  iR  10V V2
P  54W
31. Taking path ACDB R

VA  1  2  1 – 2  VB 2 2(6  x)
42. 
Hence potential at B will be +1 volt 2 6x

32. Magnitude of R  k 1 6x  12  2x ; x  3
2
Magnitude X = k 2  k 1 43. VA  V  i  4
5
33. Conceptual
3
34. By Kirchhoff second law in both loops VB  V  i  1 ; VB  VA
5
30I1  41I3  45  0 Current will flow from higher potential to
lower potential.
and 30I1  21I2  80  0

224 | CURRENT ELECTRTICITY |


ACTIVE SITE EDUTECH-9844532971

Sr | 12TH CLASS |NEET PHYSICS : VOL-1|


2E
i
44. r1  r2  R multi model
questions
 2E 
E  ir1  0 ; E  r1  0
 r1  r2  R  SINGLE ANSWER TYPE
R  r1  r2 1. Consider a current carrying wire (current
I) in the shape of a circle.
45. V  P R  2  8  4volt 1) source of emf
2) electric field produced by charges
4 accumulated on the surface of wire
i  1A ; Power in 3  i 2 R  3W
4 3) the charges just behind a given segment of
46. Kirchhoff first and second law are wire which push them just the right way by
consequences of conservation of charge repulsion
and energy respectively. 4) the charges ahead
1 1 1 2. A metal rod of length 10 cm and a rectangular
47.  
R eq R / 2 R / 2 1
cross-section of 1 cm  cm is connected
2
1 4 to a battery across opposite faces. The
 ; R ev  R / 4
R eq R resistance will be
1) maximum when the battery is connected
   
48. r   1 2  R ; r = 1.5 1
 2  across 1cm cm faces
2
49. R  760  40  800 ; 2) maximum when the battery is connected
across 10cm 1cm faces
2 1
i  3) maximum when the battery is connected
800 400
1
across 10cm  cm faces
1 2
 40
V iR 400
k   4) same irrespective of the three faces
L R 10 3. Which of the following characteristics of
electrons determines the current in a
 1102 v / m
conductor?
1 1) Drift velocity alone
50. P = I2 R ; 1  25  R ; R 
25 2) Thermal velocity alone
3) Both drift velocity and thermal velocity
18 V
51. R ;i  4) Neither drift nor thermal velocity
5 R
4. Two batteries of emf 1 and 2   2  1 
52. 18  2i  12  i
and internal resistance r1 and r2 respec-
3i  6 ; i  2A ;
tively are connected in parallel as shown
V  18  2(2) = 14V in figure.

| CURRENT ELECTRTICITY | 225


ACTIVE SITE EDUTECH-9844532971

SrSR||1212THTHCLASS
CLASS|NEET
|NEETPHYSICS
PHYSICS: VOL-1|
: VOL-1|

1 MORE THAN ONE ANSWER QUESTIONS


r1
A B 7. Temperature dependence of resistivity
2 r2   T  of semiconductors, insulators and
metals is significantly based on the following
1) Two equivalent emf eq of the two cells is
factors
between 1 and  2 , i.e., 1   eq   2 1) number of charge carriers can change with
temperature T
2) The equivalent emf eq is smaller than 1 2) time interval between two successive
collisions can depend on T
3) The eq is given by eq  1   2 always 3) length of material can be a function of T
4) mass of carriers is a function of T
4) eq is independent of internal resistance
8. Kirchhoff’s junction rule is a reflection of
r1 and r2 1) conservation of current density vector
5. Two cells of emfs approximately 5V and 2) conservation of charge
10 V are to be accurately compared using 3) the fact that the momentum with which a
a potentiometer of length 400 cm charged particle approaches a junction is
unchanged (as a vector) as the charged
1) The battery that runs the potentiometer particle leaves the junction
should have voltage of 8V
4) the fact that there is no accumulation of
2) The battery of potentiometer can have a charges at a junction
voltage of 15 V and R adjusted so that the 9. The measurement of an unknown resistance
potential drop across the wire slightly R is to be carried out using Wheatstone’s
exceeds 10 V bridge as given in the figure below. Two
3) The first portion of 50 cm of wire itself should students perform an experiment in two
have a potential drop of 10 V ways. The first students takes R 2  10
4) Potentiometer is usually used for comparing and R1  5 . The other student takes
resistance and not voltages
R 2  1000 and R1  500 . In the
6. A resistance R is to be measured using a
standard arm, both take R 3  5 .
meter bridge, student chooses the standard
resistance S to be 100 . He finds the null R2
Both find R  , R 3  10 within errors.
point at I1  2.9 cm . He is told to attempt R1
to improve the accuracy. Which of the 1) The errors of measurement of the two
following is a useful way ? students are the same
2) Errors of measurement do depend on the
1) He should measure I1 more accurately accuracy with which R 2 and R1 can be
measured
2) He should changes s to 1000  and repeat 3) If the student uses large values of
the experiment R 2 and R1 the currents through the arms
3) He should change S to 3 and repeat the will be feeble. This will make determination
of null point accurately more difficult
experiment
4) Wheatstone bridge is a very accurate
4) He should given up hoe of a more accurate instrument and has no errors of measure-
measurement with a meter bridge ment

226 | CURRENT ELECTRTICITY |


ACTIVE SITE EDUTECH-9844532971

Sr | 12TH CLASS |NEET PHYSICS : VOL-1|


10. In a meter bridge, the point D is a neutral
point (figure).
1) The meter bridge can have no other HINTS SOLUTIONS
neutral. A point for this set of resistance
2) When the jockey contacts a point on meter 1. Current per unit area (taken normal to the
wire left of D, current flows to B from the current), I/A, is called current density and is
wire denoted by j. The SI units of the current
3) When the jockey contacts a point on the density are A / m 2 . The current density is also
meter wire to the right of D, current flows
directed along E and is also a vector and the
from B to the wire through galvanometer
relationship is given by (J = SE)
4) When R is increased, the neutral point shifts
The j changes due to electric field produced by
to left
charges accumulated on the surface of wire.
Assertion and Reason Type 2. The resistance of wire is given by
11. Assertion : Potentiometer is much better l
than a voltmeter for measuring emf of R
A
cell
For greater value of R, l must be higher and A
Reason : A potentiometer draws no
current while measuring emf of a cell should be lower and it is possible only when
1) Both (A) and (R) are true and (R) is the 1
correct explanation of A. the battery is connected across 1cm    cm
 2
2) Both (A) and (R) are true but (R) is not
the correct explanation of A. (area of cross-section A).
3) (A) is true but (R) is false 3. The relationship between current and drift
4) (A) is false but (R) is true speed is given by I  ne Av d . Here, I is the
12. A : The emf of the cell in secondary circuit
current and v d is the drift velocity.. So, I  Vd
must be less than emf of cell in primary
circuit in potentiometer. Thus, only drift velocity determines the current
R : Balancing length cannot be more than a conductor.
length of potentiometer wire. 4. The equivalent emf of this combination is given by
1) Both (A) and (R) are true and (R) is the  r  r
eq  2 1 1 2
correct explanation of A. r1  r2
2) Both (A) and (R) are true but (R) is not the This suggest that the equivalent emf eq of the
correct explanation of A.
3) (A) is true but (R) is false two cells is given by 1   eq   2
4) (A) is false but (R) is true 5. In a potentiometer experiment, the emf of a cell
can be measured, if the potential drop along
the potentiometer wire is more than the emf of
the cell to be determined. Here, values of emfs
1) 2 2) 1 3) 1 4) 1 5) 2 of two cells are given as 5V and 10V, therefore,
6) 3 7) 1,2 8) 2,4 9) 2, 3
the potential drop along the potentiometer wire
10) 1,3 11)1 12)1
must be more than 10 V.

| CURRENT ELECTRTICITY | 227


ACTIVE SITE EDUTECH-9844532971

SrSR||1212THTHCLASS
CLASS|NEET
|NEETPHYSICS
PHYSICS: VOL-1|
: VOL-1|
6. The percentage error in R can be minimised by Now, according to Wheatstone bridge rule,
adjusting the balance point near the middle of R 2 R1 R
the bridge, i.e., when I1 is close to 50 cm. The   R  R3  2
R R3 R1
requires a suitable choice of S.
R R l1 l1
Since,  
S R 100  l1  100  l1
Since here, R : S : : 2.9 : 97.1 imply that the S
is nearly 33 times to that of R. In order to make
this ratio 1 : 1, it is necessary to reduced the
1
value of S nearly times i.e., nearly 3 .
33
7. Kirchhoff’s junction rule is also known as
Now putting all the values in Eq. (i) , we get
Kirchhoff’s current law which states that the
algebraic sum of the currents flowing towards R  10 for both students. Thus, we can
any point in an electric network is zero . i.e., analyse that the Wheatstone bridge is most sen-
charges are conserved in an electric network. sitive and accurate if resistance are of same
So, Kirchhoff’s junction rule is the reflection of value.
conservation of charge. Thus, the errors of measurement of the two stu-
8. The resistivity of a metallic conductor is given dents depend on the accuracy and sensitivity
of the bridge, which in turn depends on the ac-
by, e  m2 curacy with which R 2 and R1 can be mea-
ne 
where n is number of charge carriers per unit sured.
volume which can change with temperature T When R 2 and R1 are larger, the currents
and  is time interval between two successive through the arms of bridge is very weak. This
collisions which decreases with the increases can make the determination of null point accu-
of temperature. rately more difficult.
9. Given, for first student, 10. At neutral point, potential at B and neutral point
R 2  10, R 1  5, R 3  5 are same. When jockey is placed at the right
of D, the potential drop across AB, which brings
For second student, R1  500, R 3  5 the potential of point D less than that of B,
hence current flows from B to D.

228 | CURRENT ELECTRTICITY |

You might also like